Re: [obm-l] Teoria dos numeros

2020-05-22 Por tôpico Anderson Torres
Em ter., 19 de mai. de 2020 às 15:52, Israel Meireles Chrisostomo
 escreveu:
>
> Olá pessoal.Ultimamente tenho pensado em como provar que a tangente de um 
> arco racional diferente de zero é sempre irracional.

Cê diz que se r é racional então tan(r) é irracional (exceto se r=0)?

Acho que dá para ser mais arrojado e provar logo a transcendência.
Afinal, qualquer racional pode ser multiplicado e dividido até dar 1,
e as funções tangente de múltiplo arco são racionais no arco primeiro,
e se tan 1 é transcedente, acabou.

Eu consegui chegar no seguinte: Se r é real diferente  de zero e s é
inteiro diferente de zero, então ou tan(r-1/2s)  ou tan(r) é
irracional.
> Daí então eu tomo um r racional,  então ou tan(r-1/2s)  ou tan(r) é 
> irracional, se tan(r) é irracional então está provado, se por um outro lado 
> tan(r-1/2s)  é irracional então faça r= r'+1/2s e daí tem-se tan(r') é 
> irracional.O que mostra que a tangente de todo arco racional diferente de 
> zero é irracional.
> Está correto esse meu raciocínio?
> Partindo de que "se r é real diferente  de zero e s é inteiro diferente de 
> zero, então ou tan(r-1/2s)  ou tan(r) é irracional "  como posso provar isso ?
> --
> Israel Meireles Chrisostomo
>
> --
> Esta mensagem foi verificada pelo sistema de antivírus e
> acredita-se estar livre de perigo.

-- 
Esta mensagem foi verificada pelo sistema de antiv�rus e
 acredita-se estar livre de perigo.


=
Instru��es para entrar na lista, sair da lista e usar a lista em
http://www.mat.puc-rio.br/~obmlistas/obm-l.html
=


[obm-l] Teoria dos numeros

2020-05-19 Por tôpico Israel Meireles Chrisostomo
Olá pessoal.Ultimamente tenho pensado em como provar que a tangente de um
arco racional diferente de zero é sempre irracional.Eu consegui chegar no
seguinte: Se r é real diferente  de zero e s é inteiro diferente de zero,
então ou tan(r-1/2s)  ou tan(r) é irracional.
Daí então eu tomo um r racional,  então ou tan(r-1/2s)  ou tan(r) é
irracional, se tan(r) é irracional então está provado, se por um outro lado
tan(r-1/2s)  é irracional então faça r= r'+1/2s e daí tem-se tan(r') é
irracional.O que mostra que a tangente de todo arco racional diferente de
zero é irracional.
Está correto esse meu raciocínio?
Partindo de que "se r é real diferente  de zero e s é inteiro diferente de
zero, então ou tan(r-1/2s)  ou tan(r) é irracional "  como posso provar
isso ?
-- 
Israel Meireles Chrisostomo

-- 
Esta mensagem foi verificada pelo sistema de antiv�rus e
 acredita-se estar livre de perigo.



Re: [obm-l] teoria dos numeros

2020-03-30 Por tôpico Carlos Victor
 

Basta fazer (2^3-1)^2n+(2^3+1)^2n -2 e usar binômio de Newton. 

Em 28/03/2020 13:55, Israel Meireles Chrisostomo escreveu: 

> Eu sei resolver o problema abaixo,porém não sei se é a forma mais simples de 
> se fazer.Vcs poderiam por favor colocar suas soluções nos comentários dessa 
> publicação? O problema é o seguinte:
> Prove que 128 divide 49^{n} + 81^{n} -2, para todo n ≥ 1.Se possível não use 
> indução, pois eu já estou usando indução. 
> 
> -- 
> 
> Israel Meireles Chrisostomo 
> -- 
> Esta mensagem foi verificada pelo sistema de antivrus e 
> acredita-se estar livre de perigo.

 
-- 
Esta mensagem foi verificada pelo sistema de antiv�rus e
 acredita-se estar livre de perigo.



Re: [obm-l] teoria dos numeros

2020-03-30 Por tôpico Pedro José
Boa noite!

errata:
Ao invés de: 128=2^7 então 2^7| 49^{n} + 81^{n} −2<==> x= 2^7| 49^{n} +
81^{n}=2 mod2^7
128=2^7 então 2^7| 49^{n} + 81^{n} −2<==> x= 49^{n} + 81^{n}=2 mod2^7

Saudações,
PJMS

Em dom., 29 de mar. de 2020 às 14:04, Pedro José 
escreveu:

> Bom dia!
> Prove que 128 divide 49^{n} + 81^{n} −2, para todo n ≥ 1.
> 128=2^7 então 2^7| 49^{n} + 81^{n} −2<==> x= 2^7| 49^{n} + 81^{n}=2 mod2^7
> x= a + b , a= 49^n e b=81^n
> a= (64-15)^n = n(-1)^n*n*64*(15)^(n-1) + (-1)^n*15^n mod2^7; pois, os
> demais termos do binômio de Newton terão o fator (2^6)^m com m>1 que é
> côngruo 0 mod2^7.
> b= (64+17)^n = n*64*17^(n-1) + 17^n mod2^7 pelo mesmo motivo anterior.
> a+b = n*64(17^n-1 +(-1)^(n-1)*15^(n-1)) + 17^n + (-1)^n*15^n =
> (-1)^(n-1)*15^(n-1)) + 17^n + (-1)^n*15^n mod2^; pois a primeira parcela é
> côngrua a 0 mod2^7; já que o termo entre parêntesis é par.
> (16+1)^n= n*16+1 mod2^7 ,pois, (2^4)^m =0 mod2^7 para m>1
> (-1)^n*(16-1)= (-1)^n*[(-1)^(n-1)*n*16+(-1)^n]=-16n +1
> então x = a+b= 2 mod2^7 ==> 2^7 | a+b-2
>
> Saudações,
> PJMS
>
>
>
>
> Em sáb., 28 de mar. de 2020 às 14:05, Israel Meireles Chrisostomo <
> israelmchrisost...@gmail.com> escreveu:
>
>> Eu sei resolver o problema abaixo,porém não sei se é a forma mais simples
>> de se fazer.Vcs poderiam por favor colocar suas soluções nos comentários
>> dessa publicação? O problema é o seguinte:
>> Prove que 128 divide 49^{n} + 81^{n} −2, para todo n ≥ 1.Se possível não
>> use indução, pois eu já estou usando indução.
>>
>> --
>> Israel Meireles Chrisostomo
>>
>> --
>> Esta mensagem foi verificada pelo sistema de antivírus e
>> acredita-se estar livre de perigo.
>
>

-- 
Esta mensagem foi verificada pelo sistema de antiv�rus e
 acredita-se estar livre de perigo.



Re: [obm-l] teoria dos numeros

2020-03-29 Por tôpico Pedro José
Bom dia!
Prove que 128 divide 49^{n} + 81^{n} −2, para todo n ≥ 1.
128=2^7 então 2^7| 49^{n} + 81^{n} −2<==> x= 2^7| 49^{n} + 81^{n}=2 mod2^7
x= a + b , a= 49^n e b=81^n
a= (64-15)^n = n(-1)^n*n*64*(15)^(n-1) + (-1)^n*15^n mod2^7; pois, os
demais termos do binômio de Newton terão o fator (2^6)^m com m>1 que é
côngruo 0 mod2^7.
b= (64+17)^n = n*64*17^(n-1) + 17^n mod2^7 pelo mesmo motivo anterior.
a+b = n*64(17^n-1 +(-1)^(n-1)*15^(n-1)) + 17^n + (-1)^n*15^n =
(-1)^(n-1)*15^(n-1)) + 17^n + (-1)^n*15^n mod2^; pois a primeira parcela é
côngrua a 0 mod2^7; já que o termo entre parêntesis é par.
(16+1)^n= n*16+1 mod2^7 ,pois, (2^4)^m =0 mod2^7 para m>1
(-1)^n*(16-1)= (-1)^n*[(-1)^(n-1)*n*16+(-1)^n]=-16n +1
então x = a+b= 2 mod2^7 ==> 2^7 | a+b-2

Saudações,
PJMS




Em sáb., 28 de mar. de 2020 às 14:05, Israel Meireles Chrisostomo <
israelmchrisost...@gmail.com> escreveu:

> Eu sei resolver o problema abaixo,porém não sei se é a forma mais simples
> de se fazer.Vcs poderiam por favor colocar suas soluções nos comentários
> dessa publicação? O problema é o seguinte:
> Prove que 128 divide 49^{n} + 81^{n} −2, para todo n ≥ 1.Se possível não
> use indução, pois eu já estou usando indução.
>
> --
> Israel Meireles Chrisostomo
>
> --
> Esta mensagem foi verificada pelo sistema de antivírus e
> acredita-se estar livre de perigo.

-- 
Esta mensagem foi verificada pelo sistema de antiv�rus e
 acredita-se estar livre de perigo.



[obm-l] teoria dos numeros

2020-03-28 Por tôpico Israel Meireles Chrisostomo
Eu sei resolver o problema abaixo,porém não sei se é a forma mais simples
de se fazer.Vcs poderiam por favor colocar suas soluções nos comentários
dessa publicação? O problema é o seguinte:
Prove que 128 divide 49^{n} + 81^{n} −2, para todo n ≥ 1.Se possível não
use indução, pois eu já estou usando indução.

-- 
Israel Meireles Chrisostomo

-- 
Esta mensagem foi verificada pelo sistema de antiv�rus e
 acredita-se estar livre de perigo.



Re: [obm-l] Teoria dos numeros

2018-03-28 Por tôpico Pedro José
Bom dia!

Não deu para compreender. Para cada terno (k,j,w) terá apenas uma raiz em x
ou nenhuma. Mas para todo natural existe pelo menos um terno que atenda a
sua proposição.

w=x ; k=1 e j=2.

Saudações,
PJMS

Em 27 de março de 2018 22:28, Israel Meireles Chrisostomo <
israelmchrisost...@gmail.com> escreveu:

> O máximo que eu consigo é considerar uma solução que seja um número primo
>
> Em 27 de março de 2018 22:27, Israel Meireles Chrisostomo <
> israelmchrisost...@gmail.com> escreveu:
>
>> Está muito geral essas condições, achei que pudesse conseguir alguma
>> restrição a fim de resolver um outro problema, mas talvez esse caminho não
>> é muito apropriado
>>
>> Em 27 de março de 2018 22:10, Claudio Buffara 
>> escreveu:
>>
>>> O problema é só esse mesmo?
>>> Não tem nenhum contexto? Não é dada nenhuma relação entre k, j e w?
>>>
>>>
>>> 2018-03-27 21:27 GMT-03:00 Anderson Torres >> >:
>>>
 Em 27 de março de 2018 21:06, Israel Meireles Chrisostomo
  escreveu:
 > Ola pessoal eu gostaria de saber quantas são e quais são as soluções
 > naturais de (x+w)k=xj  na variável x, onde k e j  e w são naturais
 dados
 >

 (x+w)k=xj
 xk+wk=xj

 wk=xj-xk
 wk=x(j-k)

 x=wk/(j-k)


 > --
 > Israel Meireles Chrisostomo
 >
 > --
 > Esta mensagem foi verificada pelo sistema de antivírus e
 > acredita-se estar livre de perigo.

 --
 Esta mensagem foi verificada pelo sistema de antivírus e
  acredita-se estar livre de perigo.


 
 =
 Instru�ões para entrar na lista, sair da lista e usar a lista em
 http://www.mat.puc-rio.br/~obmlistas/obm-l.html
 
 =

>>>
>>>
>>> --
>>> Esta mensagem foi verificada pelo sistema de antivírus e
>>> acredita-se estar livre de perigo.
>>>
>>
>>
>>
>> --
>> Israel Meireles Chrisostomo
>>
>
>
>
> --
> Israel Meireles Chrisostomo
>
> --
> Esta mensagem foi verificada pelo sistema de antivírus e
> acredita-se estar livre de perigo.
>

-- 
Esta mensagem foi verificada pelo sistema de antiv�rus e
 acredita-se estar livre de perigo.



Re: [obm-l] Teoria dos numeros

2018-03-27 Por tôpico Israel Meireles Chrisostomo
Está muito geral essas condições, achei que pudesse conseguir alguma
restrição a fim de resolver um outro problema, mas talvez esse caminho não
é muito apropriado

Em 27 de março de 2018 22:10, Claudio Buffara 
escreveu:

> O problema é só esse mesmo?
> Não tem nenhum contexto? Não é dada nenhuma relação entre k, j e w?
>
>
> 2018-03-27 21:27 GMT-03:00 Anderson Torres :
>
>> Em 27 de março de 2018 21:06, Israel Meireles Chrisostomo
>>  escreveu:
>> > Ola pessoal eu gostaria de saber quantas são e quais são as soluções
>> > naturais de (x+w)k=xj  na variável x, onde k e j  e w são naturais dados
>> >
>>
>> (x+w)k=xj
>> xk+wk=xj
>>
>> wk=xj-xk
>> wk=x(j-k)
>>
>> x=wk/(j-k)
>>
>>
>> > --
>> > Israel Meireles Chrisostomo
>> >
>> > --
>> > Esta mensagem foi verificada pelo sistema de antivírus e
>> > acredita-se estar livre de perigo.
>>
>> --
>> Esta mensagem foi verificada pelo sistema de antivírus e
>>  acredita-se estar livre de perigo.
>>
>>
>> =
>> Instru�ões para entrar na lista, sair da lista e usar a lista em
>> http://www.mat.puc-rio.br/~obmlistas/obm-l.html
>> =
>>
>
>
> --
> Esta mensagem foi verificada pelo sistema de antivírus e
> acredita-se estar livre de perigo.
>



-- 
Israel Meireles Chrisostomo

-- 
Esta mensagem foi verificada pelo sistema de antiv�rus e
 acredita-se estar livre de perigo.



Re: [obm-l] Teoria dos numeros

2018-03-27 Por tôpico Israel Meireles Chrisostomo
O máximo que eu consigo é considerar uma solução que seja um número primo

Em 27 de março de 2018 22:27, Israel Meireles Chrisostomo <
israelmchrisost...@gmail.com> escreveu:

> Está muito geral essas condições, achei que pudesse conseguir alguma
> restrição a fim de resolver um outro problema, mas talvez esse caminho não
> é muito apropriado
>
> Em 27 de março de 2018 22:10, Claudio Buffara 
> escreveu:
>
>> O problema é só esse mesmo?
>> Não tem nenhum contexto? Não é dada nenhuma relação entre k, j e w?
>>
>>
>> 2018-03-27 21:27 GMT-03:00 Anderson Torres 
>> :
>>
>>> Em 27 de março de 2018 21:06, Israel Meireles Chrisostomo
>>>  escreveu:
>>> > Ola pessoal eu gostaria de saber quantas são e quais são as soluções
>>> > naturais de (x+w)k=xj  na variável x, onde k e j  e w são naturais
>>> dados
>>> >
>>>
>>> (x+w)k=xj
>>> xk+wk=xj
>>>
>>> wk=xj-xk
>>> wk=x(j-k)
>>>
>>> x=wk/(j-k)
>>>
>>>
>>> > --
>>> > Israel Meireles Chrisostomo
>>> >
>>> > --
>>> > Esta mensagem foi verificada pelo sistema de antivírus e
>>> > acredita-se estar livre de perigo.
>>>
>>> --
>>> Esta mensagem foi verificada pelo sistema de antivírus e
>>>  acredita-se estar livre de perigo.
>>>
>>>
>>> 
>>> =
>>> Instru�ões para entrar na lista, sair da lista e usar a lista em
>>> http://www.mat.puc-rio.br/~obmlistas/obm-l.html
>>> 
>>> =
>>>
>>
>>
>> --
>> Esta mensagem foi verificada pelo sistema de antivírus e
>> acredita-se estar livre de perigo.
>>
>
>
>
> --
> Israel Meireles Chrisostomo
>



-- 
Israel Meireles Chrisostomo

-- 
Esta mensagem foi verificada pelo sistema de antiv�rus e
 acredita-se estar livre de perigo.



Re: [obm-l] Teoria dos numeros

2018-03-27 Por tôpico Claudio Buffara
O problema é só esse mesmo?
Não tem nenhum contexto? Não é dada nenhuma relação entre k, j e w?


2018-03-27 21:27 GMT-03:00 Anderson Torres :

> Em 27 de março de 2018 21:06, Israel Meireles Chrisostomo
>  escreveu:
> > Ola pessoal eu gostaria de saber quantas são e quais são as soluções
> > naturais de (x+w)k=xj  na variável x, onde k e j  e w são naturais dados
> >
>
> (x+w)k=xj
> xk+wk=xj
>
> wk=xj-xk
> wk=x(j-k)
>
> x=wk/(j-k)
>
>
> > --
> > Israel Meireles Chrisostomo
> >
> > --
> > Esta mensagem foi verificada pelo sistema de antivírus e
> > acredita-se estar livre de perigo.
>
> --
> Esta mensagem foi verificada pelo sistema de antivírus e
>  acredita-se estar livre de perigo.
>
>
> =
> Instru�ões para entrar na lista, sair da lista e usar a lista em
> http://www.mat.puc-rio.br/~obmlistas/obm-l.html
> =
>

-- 
Esta mensagem foi verificada pelo sistema de antiv�rus e
 acredita-se estar livre de perigo.



Re: [obm-l] Teoria dos numeros

2018-03-27 Por tôpico Anderson Torres
Em 27 de março de 2018 21:06, Israel Meireles Chrisostomo
 escreveu:
> Ola pessoal eu gostaria de saber quantas são e quais são as soluções
> naturais de (x+w)k=xj  na variável x, onde k e j  e w são naturais dados
>

(x+w)k=xj
xk+wk=xj

wk=xj-xk
wk=x(j-k)

x=wk/(j-k)


> --
> Israel Meireles Chrisostomo
>
> --
> Esta mensagem foi verificada pelo sistema de antivírus e
> acredita-se estar livre de perigo.

-- 
Esta mensagem foi verificada pelo sistema de antiv�rus e
 acredita-se estar livre de perigo.


=
Instru��es para entrar na lista, sair da lista e usar a lista em
http://www.mat.puc-rio.br/~obmlistas/obm-l.html
=


[obm-l] Teoria dos numeros

2018-03-27 Por tôpico Israel Meireles Chrisostomo
Ola pessoal eu gostaria de saber quantas são e quais são as soluções
naturais de (x+w)k=xj  na variável x, onde k e j  e w são naturais dados

-- 
Israel Meireles Chrisostomo

-- 
Esta mensagem foi verificada pelo sistema de antiv�rus e
 acredita-se estar livre de perigo.



Re: [obm-l] Teoria dos numeros

2016-12-22 Por tôpico Pedro José
Bom dia!

x= 0  y= 1 e z= 1 ; a = -1, b=-1 e c=-1

-1.0 + -1.1 + -1.1 = -1 + 0 -1 (V) atende a

1 + 1 =1 > = 0 +1 +1 (V) atende b.

-1 não é soma de três quadrados de inteiros.

Tem que ter mais restrições.

Saudações,
PJMS



Em 20 de dezembro de 2016 19:08, Gabriel Tostes 
escreveu:

> A,b,c,X,y,z inteiros tais que
> a) ax^2+by^2+cz^2=abc +2xyz - 1
> B) ab+bc+ca>=x^2+y^2+z^2
>
> Provar que a,b,c são somas de 3 quadrados de inteiros
>
>
>
> --
> Esta mensagem foi verificada pelo sistema de antivírus e
>  acredita-se estar livre de perigo.
>
>
> =
> Instruções para entrar na lista, sair da lista e usar a lista em
> http://www.mat.puc-rio.br/~obmlistas/obm-l.html
> =
>

-- 
Esta mensagem foi verificada pelo sistema de antiv�rus e
 acredita-se estar livre de perigo.



[obm-l] Teoria dos numeros

2016-12-20 Por tôpico Gabriel Tostes
A,b,c,X,y,z inteiros tais que
a) ax^2+by^2+cz^2=abc +2xyz - 1
B) ab+bc+ca>=x^2+y^2+z^2

Provar que a,b,c são somas de 3 quadrados de inteiros



-- 
Esta mensagem foi verificada pelo sistema de antiv�rus e
 acredita-se estar livre de perigo.


=
Instru��es para entrar na lista, sair da lista e usar a lista em
http://www.mat.puc-rio.br/~obmlistas/obm-l.html
=


Re: [obm-l] Teoria dos numeros

2013-11-19 Por tôpico terence thirteen
Ivan Niven and Herbert Zuckermann.

Fala de tudo que é possível ser dito de forma elementar, e é uma leitura
divertida por si mesma.


Em 28 de outubro de 2013 20:08, Hermann ilhadepaqu...@bol.com.br escreveu:

  Veja na livraria da SBM tem uns muito bons

 - Original Message -
 *From:* sergio marinho smarinh...@yahoo.com.br
 *To:* obm-l@mat.puc-rio.br
 *Sent:* Monday, October 28, 2013 4:54 PM
 *Subject:* Re: [obm-l] Teoria dos numeros


 Vc poderia me indicar excelentes livros de Teoria dos números e Análise
 combinatória?

 Grato.  Sérgio Soares.


  Em Sábado, 3 de Agosto de 2013 16:47, marcone augusto araújo borges 
 marconeborge...@hotmail.com escreveu:
   Seja n uma soma de dois numeros triangulares (a^2 + a)/2 e (b^2 + b)/2.
   Mostre que 4n + 1 é uma soma de dois quadrados em termos de a e
 b.

 --
 Esta mensagem foi verificada pelo sistema de antivírus e
 acredita-se estar livre de perigo.



 --
 Esta mensagem foi verificada pelo sistema de antivírus e
 acredita-se estar livre de perigo.


 --
 Esta mensagem foi verificada pelo sistema de antivírus e
 acredita-se estar livre de perigo.




-- 
/**/
神が祝福

Torres

-- 
Esta mensagem foi verificada pelo sistema de antiv�rus e
 acredita-se estar livre de perigo.



Re: [obm-l] Teoria dos numeros

2013-10-28 Por tôpico sergio marinho


Vc poderia me indicar excelentes livros de Teoria dos números e Análise 
combinatória?

Grato.  Sérgio Soares.




Em Sábado, 3 de Agosto de 2013 16:47, marcone augusto araújo borges 
marconeborge...@hotmail.com escreveu:
 
 
Seja n uma soma de dois numeros triangulares (a^2 + a)/2 e (b^2 + b)/2. 
Mostre que 4n + 1 é uma soma de dois quadrados em termos de a e b.
-- 
Esta mensagem foi verificada pelo sistema de antivírus e 
acredita-se estar livre de perigo. 
-- 
Esta mensagem foi verificada pelo sistema de antivírus e
 acredita-se estar livre de perigo.



Re: [obm-l] Teoria dos numeros

2013-10-28 Por tôpico Hermann
Veja na livraria da SBM tem uns muito bons
  - Original Message - 
  From: sergio marinho 
  To: obm-l@mat.puc-rio.br 
  Sent: Monday, October 28, 2013 4:54 PM
  Subject: Re: [obm-l] Teoria dos numeros




  Vc poderia me indicar excelentes livros de Teoria dos números e Análise 
combinatória?


  Grato.  Sérgio Soares.



  Em Sábado, 3 de Agosto de 2013 16:47, marcone augusto araújo borges 
marconeborge...@hotmail.com escreveu:

  Seja n uma soma de dois numeros triangulares (a^2 + a)/2 e (b^2 + b)/2.
   
   
   
   
   
   
   
   
   
  Mostre que 4n + 1 é uma soma de dois quadrados em termos de a e b.

  -- 
  Esta mensagem foi verificada pelo sistema de antivírus e 
  acredita-se estar livre de perigo. 




  -- 
  Esta mensagem foi verificada pelo sistema de antivírus e 
  acredita-se estar livre de perigo. 
-- 
Esta mensagem foi verificada pelo sistema de antivírus e
 acredita-se estar livre de perigo.



[obm-l] Teoria dos numeros

2013-08-03 Por tôpico marcone augusto araújo borges
Seja n uma soma de dois numeros triangulares (a^2 + a)/2 e (b^2 + b)/2.



 
 



  


  


  


  


 
 



  


  


  


  

Mostre que 4n + 1 é uma soma de dois quadrados em termos de a e b.  
  
-- 
Esta mensagem foi verificada pelo sistema de antivírus e
 acredita-se estar livre de perigo.



Re: [obm-l] Teoria dos numeros

2013-08-03 Por tôpico Nehab

Oi, Marcone.

Ora, você quer que a soma de dois quadrados dê 2a^2 + 2b^2 + 2a + 2b + 1.
O a^2 e o b^2 saem de coisas do tipo (a + b +...)^2 e (a - b +...)^2.
Para se livrar do 2ab que aparece nessa coisas, você precisa de um +2ab 
e de um -2ab...
Dai, botando os neurônios para esquentar um pouquinho, fica fácil: (a - 
b)^2 + (a + b + 1)^2.


Abraços,
Nehab


On 03/08/2013 16:32, marcone augusto araújo borges wrote:

Seja n uma soma de dois numeros triangulares (a^2 + a)/2 e (b^2 + b)/2.
[Upload Photo to Facebook]
[Google+]
[Twitt]
[Send by Gmail]
[Upload Video to Facebook]
[Google+]
[Twitt]
[Send by Gmail]
Mostre que 4n + 1 é uma soma de dois quadrados em termos de a e b.

--
Esta mensagem foi verificada pelo sistema de antivírus e
acredita-se estar livre de perigo. 



--
Esta mensagem foi verificada pelo sistema de antivírus e
acredita-se estar livre de perigo.



[obm-l] teoria dos numeros

2013-05-10 Por tôpico valdir soares
Ola pessoal,

Gostaria de saber, como fazer o problema abaixo :

Determine n entre 100 e 1000 , tal que   ( 2+ 2^n)/n  eh tambem inteiro .

Obrigado


Re: [obm-l] teoria dos numeros

2013-05-10 Por tôpico terence thirteen
Aprenda um pouco de inglês:

http://ohkawa.cc.it-hiroshima.ac.jp/www.kalva.demon.co.uk/apmo/asoln/asol972.html


Em 10 de maio de 2013 06:48, valdir soares valdir.soa...@oi.com.brescreveu:

 Ola pessoal,

 Gostaria de saber, como fazer o problema abaixo :

 Determine n entre 100 e 1000 , tal que   ( 2+ 2^n)/n  eh tambem inteiro .

 Obrigado




-- 
/**/
神が祝福

Torres


Re: [obm-l] teoria dos numeros

2013-05-10 Por tôpico Bernardo Freitas Paulo da Costa
2013/5/10 terence thirteen peterdirich...@gmail.com:
 Aprenda um pouco de inglês:

 http://ohkawa.cc.it-hiroshima.ac.jp/www.kalva.demon.co.uk/apmo/asoln/asol972.html


 Em 10 de maio de 2013 06:48, valdir soares valdir.soa...@oi.com.br
 escreveu:

 Ola pessoal,

 Gostaria de saber, como fazer o problema abaixo :

 Determine n entre 100 e 1000 , tal que   ( 2+ 2^n)/n  eh tambem inteiro .

Braço por braço (a solução contando os primos e verificando que 2 é
primitiva módulo p, e depois mais braço para p, 2p, pq, 2pq), dá pra
rodar todos esses números. E (ao contrário do kalva) eu usei a
calculadora do linux em linha de comando:

$ bc
define r(n) { n ; return (2 + 2^n) % n }
for (i = 100; i = 1000; i++) r(i)

Depois, com a resposta na mão, você apenas verifica que dá certo ;-)
-- 
Bernardo Freitas Paulo da Costa

=
Instruções para entrar na lista, sair da lista e usar a lista em
http://www.mat.puc-rio.br/~obmlistas/obm-l.html
=


[obm-l] Teoria dos numeros!

2007-10-03 Por tôpico jose silva

  Se possivel, gostaria de contar com a ajuda de voces, para resolver este 
problema. 
 
  Mostre que se g e uma raiz impar primitiva de p^m (pelevado a m) com p 
maior que 2, entao g e uma raiz primitiva de 2p^m (2p elevado a m).
 
  Desde ja, muito obrigado!
  jccardosos. 
_
Conheça o Windows Live Spaces, a rede de relacionamentos conectada ao Messenger!
http://spaces.live.com/signup.aspx

[obm-l] Teoria dos Numeros

2007-10-01 Por tôpico Marcelo Salhab Brogliato
Olá a todos,

estou meio sumido, mas acho q ainda sim posso mandar uma questaozinha q nao
consegui resolver..
alias, nem sei c tem solucao...

Determine Sum{i=1 ... n} ( k mod i )

apenas para relatar a origem.. eh um problema de programacao, onde 1 = k, n
= 10^9...
como nao encontrei uma solucao fechada, procurei algumas alternativas, e
acabei encontrando algumas propriedades...
por exemplo:
Seja f(n) = Sum{i=1...n} {k mod i}.
Entao, se n  k, temos que: f(n) = f(k) + k*(n-k)
é possível mostrar que f(k) = f([k/2]) + g(k), onde [k/2] é o piso de k/2, e
g(k) é conhecido e possui uma forma fechada..

minha duvida é: existe uma forma fechada para este somatorio??

obrigado,
Salhab


RE: RES: [obm-l] Teoria dos numeros

2007-06-14 Por tôpico Rhilbert Rivera

 
Prezados Paulo e Artur.
Paulo acredito que você não cometeu nenhum erro de cálculo.
 
Analise o que fiz.Primeiro, pensando na decomposição de um fatorial em fatores 
primos, escrevi 10200 nas bases 2, 3 e 7 respectivamente: 
10200 = (1001011)2
10200 = (1111)3
10200 = (41511)7
Determinano as potências de 2, 3 e 7 na decomposição de  10200! em fatores 
primos: 10200-(1+1+1+1+1+1+1+1)/(2-1) = 10192 (potência do 
2);10200-(1+1+1+2+2+2+2+1)/(3-1) = 5094 ( potência do 
3);10200-(4+1+5+1+1)/(7-1) = 1698  ( potência do 7) Fazendo a decomposição 
dessas potências em fatores primos: 
10192 = 24 . 72 . 13  

5094 = 2 . 32. 283
1698 = 2. 3. 283  
 
Podemos ainda escrever os valores dessas potências como:
 
10192 = 6. 6. 283 + 4  =1698 . 6 + 4 
5094 = 3. 6. 283 = 1698 . 3
1698 = 6 . 283 = 1698 . 1
Logo, o valor de  é n = 1698
 
[[ ]]'s
 Subject: RES: [obm-l] Teoria dos numeros Date: Tue, 12 Jun 2007 13:20:44 
 -0300 From: [EMAIL PROTECTED] To: obm-l@mat.puc-rio.br  Obrigado Paulo 
 Abraços Artur  -Mensagem original- De: [EMAIL PROTECTED] 
 [mailto:[EMAIL PROTECTED] nome de Paulo Santa Rita Enviada em: terça-feira, 
 12 de junho de 2007 11:24 Para: obm-l@mat.puc-rio.br Assunto: Re: [obm-l] 
 Teoria dos numeros   Ola Carissimo Artur e demais colegas desta lista ... 
 OBM-L,  E facil ver que 7^4  10200  7^5. Assim, basta considerar ate 7^4. 
 De 7 ate 10199 temos 10199 = 7 + (A-1)*7 = A = 1457 multiplos de 7. 
 Considerando os multiplos de 49 teriamos 10.192 = 49 + (B-1)*49 = B=208 
 multiplos de 49 e com o mesmo raciocinio achamos 29 multiplos de 343(=7^3) e 
 4 multiplos de 2401 (= 7^4). Logo, o total de fatores 7 em 10200 ! e A + B + 
 C + D = 1698.  Como de 1 ate 10200 existem 1 numero par ( divisivel por 2 ) 
 a cada dois numeros segue que ha mais que 10200 / 2 = 5100 fatores 2 e, 
 alem disso, 5100  3*1698 = 5094. Igualmente, como de 1 ate 10200 existem 1 
 numero divisivel por 3 a cada tres numeros segue que ha mais que 10200 / 3 = 
 3400 fatores 3 e, alem disso, 3400  2*1698 = 3396  Segue que N = 1698 e o 
 numero procurado.  Esta e uma solucao PARA ATROPELAR A QUESTAO, isto e, 
 resolucao truculenta tipo forca bruta. Nao ha inteligencia aqui. Eu 
 precisaria ficar receptivo para receber ideias bonitas mas estou sem tempo. 
  Um Abracao Paulo Santa Rita 3,0A20,120607  Em tempo : por favor, 
 verifique se nao cometi algum erro de calculo. O raciocinio e correto, eu 
 garanto  Em 11/06/07, Artur Costa Steiner[EMAIL PROTECTED] escreveu:  
   Estou tentando achar uma solucoa para o seguinte, mas ainda nao 
 consegui:   Encontrar o mair valor do ineiro n=0 tal que 
 (10200!)/(504^n) seja inteiro.  Nos temos que 504 = 2^3 * 3^2 * 7, assim, o 
 quociente sera inteiro  enquanto 10200! contiver os primos 2, 3 e 7 com 
 expoentes no maximo de 3n ,  2n e n, respectivamente. Mas nao sei se hah 
 uma forma facil de fazer isso.   Obrigado  Artur 
 = 
 Instruções para entrar na lista, sair da lista e usar a lista em 
 http://www.mat.puc-rio.br/~nicolau/olimp/obm-l.html 
 =  
 = 
 Instruções para entrar na lista, sair da lista e usar a lista em 
 http://www.mat.puc-rio.br/~nicolau/olimp/obm-l.html 
 =
_
Receba GRÁTIS as mensagens do Messenger no seu celular quando você estiver 
offline. Conheça  o MSN Mobile!
http://mobile.live.com/signup/signup2.aspx?lc=pt-br

Re: [obm-l] Teoria dos numeros

2007-06-12 Por tôpico Paulo Santa Rita

Ola Carissimo Artur e demais
colegas desta lista ... OBM-L,

E facil ver que 7^4  10200  7^5. Assim, basta considerar ate 7^4. De
7 ate  10199 temos 10199 = 7 + (A-1)*7  = A = 1457 multiplos de 7.
Considerando os multiplos de 49 teriamos 10.192 = 49 + (B-1)*49 =
B=208 multiplos de 49 e com o mesmo raciocinio achamos 29 multiplos de
343(=7^3) e 4 multiplos de 2401 (= 7^4). Logo, o total de fatores 7 em
10200 ! e A + B + C + D = 1698.

Como de 1 ate 10200 existem 1 numero par ( divisivel por 2 ) a cada
dois numeros segue que ha mais que 10200 / 2 = 5100 fatores 2 e, alem
disso, 5100  3*1698 = 5094. Igualmente, como de 1 ate 10200 existem 1
numero divisivel por 3 a cada tres numeros segue que ha mais que 10200
/ 3 = 3400 fatores 3 e, alem disso, 3400  2*1698 = 3396

Segue que N = 1698 e o numero procurado.

Esta e uma solucao PARA ATROPELAR A QUESTAO, isto e, resolucao
truculenta tipo forca bruta. Nao ha inteligencia aqui. Eu precisaria
ficar receptivo para receber ideias bonitas mas estou sem tempo.

Um Abracao
Paulo Santa Rita
3,0A20,120607

Em tempo : por favor, verifique se nao cometi algum erro de calculo. O
raciocinio e correto, eu garanto

Em 11/06/07, Artur Costa Steiner[EMAIL PROTECTED] escreveu:



Estou tentando achar uma solucoa para o seguinte, mas ainda nao consegui:

Encontrar o mair valor do ineiro n=0 tal que (10200!)/(504^n) seja inteiro.
Nos temos que 504 = 2^3  * 3^2 *  7, assim, o quociente sera inteiro
enquanto 10200! contiver os primos 2, 3  e 7 com expoentes no maximo de 3n ,
2n e n, respectivamente. Mas nao sei se hah uma forma facil de fazer isso.

Obrigado
Artur

=
Instruções para entrar na lista, sair da lista e usar a lista em
http://www.mat.puc-rio.br/~nicolau/olimp/obm-l.html
=


RE: RES: [obm-l] Teoria dos numeros

2007-06-12 Por tôpico Qwert Smith

Estranho...meu email resposta com outra conta nao apareceu na lista.

A resposta e a mesma, e eu usei o metodo de somar a parte inteira de
[n/p] + [n/p^2] + [n/p^3] + ...

A parte interessante e mostrar que o expoente de 7 e mesmo a resposta ja que
se o expoente de 7 em N! for n entao o expoente de 2 vai ser sempre PELO 
MENOS 3n e o expoente de 3 vai ser sempre PELO MENOS 2n.


Eu nao sei mostrar isso com muita formalidade nao.  O raciocinio que eu usei 
foi assim:
O expoente de 7 aumenta em 1 cada vez que N aumenta em 7, logo N aumentou em 
mais que 6 e portanto o expoente de 2 aumentou pelo menos de 3 e o expoente 
de 3 aumentou de pelo menos 2.




From: Artur Costa Steiner [EMAIL PROTECTED]
Reply-To: obm-l@mat.puc-rio.br
To: obm-l@mat.puc-rio.br
Subject: RES: [obm-l] Teoria dos numeros
Date: Tue, 12 Jun 2007 13:20:44 -0300

Obrigado Paulo
Abraços
Artur

-Mensagem original-
De: [EMAIL PROTECTED] [mailto:[EMAIL PROTECTED]
nome de Paulo Santa Rita
Enviada em: terça-feira, 12 de junho de 2007 11:24
Para: obm-l@mat.puc-rio.br
Assunto: Re: [obm-l] Teoria dos numeros


Ola Carissimo Artur e demais
colegas desta lista ... OBM-L,

E facil ver que 7^4  10200  7^5. Assim, basta considerar ate 7^4. De
7 ate  10199 temos 10199 = 7 + (A-1)*7  = A = 1457 multiplos de 7.
Considerando os multiplos de 49 teriamos 10.192 = 49 + (B-1)*49 =
B=208 multiplos de 49 e com o mesmo raciocinio achamos 29 multiplos de
343(=7^3) e 4 multiplos de 2401 (= 7^4). Logo, o total de fatores 7 em
10200 ! e A + B + C + D = 1698.

Como de 1 ate 10200 existem 1 numero par ( divisivel por 2 ) a cada
dois numeros segue que ha mais que 10200 / 2 = 5100 fatores 2 e, alem
disso, 5100  3*1698 = 5094. Igualmente, como de 1 ate 10200 existem 1
numero divisivel por 3 a cada tres numeros segue que ha mais que 10200
/ 3 = 3400 fatores 3 e, alem disso, 3400  2*1698 = 3396

Segue que N = 1698 e o numero procurado.

Esta e uma solucao PARA ATROPELAR A QUESTAO, isto e, resolucao
truculenta tipo forca bruta. Nao ha inteligencia aqui. Eu precisaria
ficar receptivo para receber ideias bonitas mas estou sem tempo.

Um Abracao
Paulo Santa Rita
3,0A20,120607

Em tempo : por favor, verifique se nao cometi algum erro de calculo. O
raciocinio e correto, eu garanto

Em 11/06/07, Artur Costa Steiner[EMAIL PROTECTED] escreveu:


 Estou tentando achar uma solucoa para o seguinte, mas ainda nao 
consegui:


 Encontrar o mair valor do ineiro n=0 tal que (10200!)/(504^n) seja 
inteiro.

 Nos temos que 504 = 2^3  * 3^2 *  7, assim, o quociente sera inteiro
 enquanto 10200! contiver os primos 2, 3  e 7 com expoentes no maximo de 
3n ,
 2n e n, respectivamente. Mas nao sei se hah uma forma facil de fazer 
isso.


 Obrigado
 Artur
=
Instruções para entrar na lista, sair da lista e usar a lista em
http://www.mat.puc-rio.br/~nicolau/olimp/obm-l.html
=

=
Instruções para entrar na lista, sair da lista e usar a lista em
http://www.mat.puc-rio.br/~nicolau/olimp/obm-l.html
=


_
Need a break? Find your escape route with Live Search Maps. 
http://maps.live.com/default.aspx?ss=Restaurants~Hotels~Amusement%20Parkcp=33.832922~-117.915659style=rlvl=13tilt=-90dir=0alt=-1000scene=1118863encType=1FORM=MGAC01


=
Instruções para entrar na lista, sair da lista e usar a lista em
http://www.mat.puc-rio.br/~nicolau/olimp/obm-l.html
=


[obm-l] Teoria dos numeros

2007-06-11 Por tôpico Artur Costa Steiner
Estou tentando achar uma solucoa para o seguinte, mas ainda nao consegui:
 
Encontrar o mair valor do ineiro n=0 tal que (10200!)/(504^n) seja inteiro. 
Nos temos que 504 = 2^3  * 3^2 *  7, assim, o quociente sera inteiro enquanto 
10200! contiver os primos 2, 3  e 7 com expoentes no maximo de 3n , 2n e n, 
respectivamente. Mas nao sei se hah uma forma facil de fazer isso.
 
Obrigado
Artur


Re:[obm-l] Teoria dos numeros?

2006-08-02 Por tôpico claudio\.buffara





De:
[EMAIL PROTECTED]




Para:
obm-l@mat.puc-rio.br




Cópia:





Data:
Tue, 01 Aug 2006 14:37:56 -0400




Assunto:
[obm-l] Teoria dos numeros?
 Liste todos os pares (m,n) para os quais 2^m + 3^n e um quadrado perfeito.
 
Estou supondo que m e n são inteiros não-negativos.

Por inspeção obtemos as soluções:
m = 0, n = 1 == 2^0 + 3^1 = 4
m = 3, n = 0 == 2^3 + 3^0 = 9
Aliás, estas são as únicas soluções com m = 0 ou n = 0.

Quem conhece o triângulo pitagórico (3,4,5) também acha rápido:
m = 4, n = 2 == 2^4 + 3^2 = 25

Alguns casos podem ser eliminados via congruências.

Por exemplo, se m =1 e n é ímpar, então 2^m + 3^n é ímpar.
Além disso, n ímpar == 3^n == 3 (mod 8). 
Logo:
m = 1 == 2^m + 3^n ==2 + 3 ==5 (mod 8).
m = 2 == 2^m + 3^n == 4+ 3 == 7 (mod 8)
m = 3 == 2^m + 3^n == 0 + 3 == 3 (mod 8)
No entanto,o quadrado de um ímparé sempre == 1 (mod 8).
Conclusão: a única solução com n ímpar é m = 0, n = 1.

***

n é par (n = 2p,p = 0) ==
2^m + 3^(2p) = a^2 ==
2^m = (a - 3^p)(a + 3^p) ==
a - 3^p = 2^k e a + 3^p = 2^(m-k), com m  2k ==
2*3^p = 2^(m-k) - 2^k = 2^k*(2^(m-2k) - 1) ==
3^p = 2^(k-1)*(2^(m-2k) - 1) ==
k = 1 (fatoração única em Z) ==
3^p = 2^(m-2) - 1 ==
m = 3

m = 3 == 3^p = 1 == p = 0 == n = 0 == (m,n) = (3,0)
m = 4 == 3^p = 3 == p = 1 == n = 2 == (m,n) = (4,2)
m = 5 ==
(fazendo q = m-2, de modo que q = 3)
2^q = 3^p + 1 ==
3^p ==-1 == 7(mod 8) ==
não há soluções neste caso, pois 3^p == 1 ou 3 (mod 8), conforme p seja par ou ímpar

Logo, as únicas soluções são (0,1), (3,0) e (4,2).

[]s,
Claudio.


[obm-l] Teoria dos numeros?

2006-08-01 Por tôpico Qwert Smith

Liste todos os pares (m,n) para os quais 2^m + 3^n e um quadrado perfeito.


=
Instruções para entrar na lista, sair da lista e usar a lista em
http://www.mat.puc-rio.br/~nicolau/olimp/obm-l.html
=


Re: [obm-l] Teoria dos numeros?

2006-08-01 Por tôpico Marcelo Salhab Brogliato

Olá,
uma parte da resposta seria:

(2a, 0)
(0, 2a)
onde a pertence aos inteiros positivos

(4, 2) tb é...
to tentando achar algum padrao pra isso... pq algebricamente eu nao consegui 
resolver...


espero ter ajudado em algo
abraços,
Salhab



- Original Message - 
From: Qwert Smith [EMAIL PROTECTED]

To: obm-l@mat.puc-rio.br
Sent: Tuesday, August 01, 2006 3:37 PM
Subject: [obm-l] Teoria dos numeros?



Liste todos os pares (m,n) para os quais 2^m + 3^n e um quadrado perfeito.


=
Instruções para entrar na lista, sair da lista e usar a lista em
http://www.mat.puc-rio.br/~nicolau/olimp/obm-l.html
=


--
No virus found in this incoming message.
Checked by AVG Free Edition.
Version: 7.1.394 / Virus Database: 268.10.5/405 - Release Date: 1/8/2006




=
Instruções para entrar na lista, sair da lista e usar a lista em
http://www.mat.puc-rio.br/~nicolau/olimp/obm-l.html
=


Re: [obm-l] Teoria dos numeros?

2006-08-01 Por tôpico Manuel Garcia
Boa noite,Acho que há alguns problemas com a resposta parcial abaixoOn 8/1/06, Marcelo Salhab Brogliato [EMAIL PROTECTED]
 wrote:Olá,uma parte da resposta seria:(2a, 0)(0, 2a)onde a pertence aos inteiros positivos
Nenhum desses pares é solução, repare que 2^0=3^0=1.O problema proposto pode ser resolvido com uma tediosa análise de congruências nada emocionante, salvo algum engano as únicas soluções são 
m=3, n=0 (nesse caso q=3)m=4, n=2 (q=5)m=0, n=1 (q=2)Manuel Garcia
(4, 2) tb é...to tentando achar algum padrao pra isso... pq algebricamente eu nao conseguiresolver...espero ter ajudado em algoabraços,Salhab- Original Message -From: Qwert Smith 
[EMAIL PROTECTED]To: obm-l@mat.puc-rio.brSent: Tuesday, August 01, 2006 3:37 PMSubject: [obm-l] Teoria dos numeros?
 Liste todos os pares (m,n) para os quais 2^m + 3^n e um quadrado perfeito. = Instruções para entrar na lista, sair da lista e usar a lista em
 http://www.mat.puc-rio.br/~nicolau/olimp/obm-l.html =
 -- No virus found in this incoming message. Checked by AVG Free Edition. Version: 7.1.394 / Virus Database: 268.10.5/405 - Release Date: 1/8/2006=
Instruções para entrar na lista, sair da lista e usar a lista emhttp://www.mat.puc-rio.br/~nicolau/olimp/obm-l.html=



Re: [obm-l] Teoria dos Numeros[off - topic]

2006-01-28 Por tôpico Danilo Nascimento
Ola Henrique,(x+1)^3-x^3=y^2 -- desenvolva o cubo perfeito.  3x^2+6x+1=y^2 --- multiplique tudo por 4  12x^2+24x+4 = 4y^2--- faça o 4=3+1  12x^2+24x+3=4y^2-1  3(4x^2+8x+1)=(2y-1)(2y+1)  2(2x+1)^2=(2y-1)(2y+1)  Dai use que (2y-1)(2y+1) sao primos entre si.  Veja q letra b) nao pode ocorrer porque ficaria 3c^2+2=d^2  dai eh so vc olhar a expressao no mod 3. como todo quadrado eh congruente a 0 ou 1 mod3. logo nao pode ser. Henrique Rennó [EMAIL PROTECTED] escreveu:  Olá Klaus e Carlos VictorFiz duas observações nesta questão e gostaria que vocês me ajudassem.Klaus, eu tinha lhe enviado por e-mail um arquivo .doc do word com umapossível solução para um exercíci!
o que
 fosse havia postado sobre acharum ângulo de um triângulo formado pelos lados dos polígonos regularesde 3,4 e 6 lados inscritos num círculo. Você recebeu??? Se não, meavise que te envio novamente.Abraços!!! (x+1)^3 - x^3 = y^2 , onde 3(2x+1)^2 = (2y-1)(2y+1) . Observe queNão entendi essa expressão: 3(2x+1)^2 = (2y-1)(2y+1). O que foipensado para formar ela??? podemos concluir que : a) Ou 2y-1 = a^2 e 2y+1 = 3b^2 b) Ou 2y-1 = 3c^2 e 2y+1 = d^2 . Observe que 3b^2 = a^2 +2 é a única que pode ocorrer e, como a é ímpar , podemos escrever a = 2t +1 e 4y = 2(a^2+1) implicando y = t^2 + (t+1)^2 , ok ? OBS : (1) Esta questão se encontra no Livro POWER PLAY de EDWARD J. BARBEAU da MAA ; inclusive com a solução acima (2) O interessante é que para 3x^2+3x +1 =y^2 tem para solução geral : x1!
 =
 4y+7x+3 e y1 = 7y+12x+6 com x e y conhecidos . Exemplo : x1 = 104 e y1 =181 ; Lindo não é ? []´s Carlos Victor At 20:23 24/1/2006, Klaus Ferraz wrote:Esse enunciado não deveria ser: Mostre que "se" a diferençaPorque, por exemplo, 5^3 - 4^3 = 125 - 64 = 61. Não existe raizquadrada inteira de 61. Mostre que a diferença entre os cubos de dois numeros inteiros consecutivos é igual ao quadrado de um inteiro, entao esse inteiro é igual a soma dos quadrados ! de dois inteiros consecutivos. Ex: 8^3-7^3=169. 2^2+3^2=13. Grato. Yahoo! doce lar. Faça do Yahoo! sua homepage.  Yahoo! doce lar. Faça do Yahoo! sua homepage.  No virus found in this incoming message. C!
hecked by
 AVG Free Edition. Version: 7.1.375 / Virus Database: 267.14.22/238 - Release Date: 23/1/2006--Henrique=Instruções para entrar na lista, sair da lista e usar a lista emhttp://www.mat.puc-rio.br/~nicolau/olimp/obm-l.html=  __Faça ligações para outros computadores com o novo Yahoo! Messenger http://br.beta.messenger.yahoo.com/ 

Re: [obm-l] Teoria dos Numeros[off - topic]

2006-01-28 Por tôpico Henrique Rennó
Olá Danilo!!!

Agradeço a resposta. Acho que tem umas correções no desenvolvimento da
expressão a serem feitas.

Klaus,

Os polígonos são de 4, 6 e 10 lados e não 3, 4 e 6.

 (x+1)^3-x^3=y^2 -- desenvolva o cubo perfeito.
 3x^2+6x+1=y^2 --- multiplique tudo por 4
3x^2 + 3x + 1 = y^2 -- não 6x
 12x^2+24x+4 = 4y^2--- faça o 4=3+1
12x^2 + 12x + 4 = 4y^2 -- não 24x
 12x^2+24x+3=4y^2-1
mesmo do anterior
 3(4x^2+8x+1)=(2y-1)(2y+1)
3(4x^2 + 4x + 1) = (2y-1)(2y+1) -- não 8x
 2(2x+1)^2=(2y-1)(2y+1)
3(2x+1)^2 = (2y-1)(2y+1) -- não 2(2x+1)^2

Mas não entendi a seguinte parte:

  O interessante é que para 3x^2+3x +1 =y^2 tem para solução
  geral :
 
  x1! = 4y+7x+3 e y1 = 7y+12x+6 com x e y conhecidos . Exemplo :
  x1 = 104 e y1 =181

Abraços,

--
Henrique

=
Instruções para entrar na lista, sair da lista e usar a lista em
http://www.mat.puc-rio.br/~nicolau/olimp/obm-l.html
=


Re:[obm-l] Teoria dos Numeros II

2006-01-28 Por tôpico Luiz H\. Barbosa

2) Para quais inteiros n, 18(n^2+3) é cubo perfeito? 

=
Vou resolver esse sem nenhuma ideia esperta:
Se 18(n^2+3) é cubo perfeito , então:

18(n^2+3) = x^3 e x0
3.3.2(n^2+3) = x.x^2
Como x é inteiro , temos varios casos:
x=2,x=3,x=6, x=9 e x=18 e depois x=(n^2+3),x=2(n^2+3),x=3(n^2+3),x=6(n^2+3) ,x=9(n^2+3) e x=18(n^2+3) .
Se analizar ai , vai ver que o unico x possivel é x=6 , assim:
3(n^2+3) = 36 -- n = +/-3.

[]'s
Luiz H. Barbosa 



Re:[obm-l] Teoria dos Numeros II

2006-01-28 Por tôpico Danilo Nascimento
ou  veja que 18(n^2+3)=(n+3)^3-(n-3)^3  logo pelo ultimo teorema de fermat, x^n=y^n+z^n, em particular para n=3 a equacao nao possui solucao. dessa forma n+3=0 ou n-3=0 logo n= -+3."Luiz H. Barbosa" [EMAIL PROTECTED] escreveu:  2) Para quais inteiros n, 18(n^2+3) é cubo perfeito? =  Vou resolver esse sem nenhuma ideia esperta:  Se 18(n^2+3) é cubo perfeito , então:18(n^2+3) = x^3 e x0  3.3.2(n^2+3) = x.x^2  Como x é inteiro , temos varios casos:  x=2,x=3,x=6, x=9 e x=18 e depois x=(n^2+3),x=2(n^2+3),x=3(n^2+3),x=6(n^2+3) ,x=9(n^2+3) e x=18(n^2+3) .  Se analizar ai , vai ver que o unico x possivel é x=6 , assim!
: 
 3(n^2+3) = 36 -- n = +/-3.[]'s  Luiz H. Barbosa 
		 
Yahoo! doce lar. Faça do Yahoo! sua homepage.

Re: [obm-l] Teoria dos Numeros

2006-01-26 Por tôpico Carlos Victor


Olá Klauss ,
(x+1)^3 - x^3 = y^2 , onde 3(2x+1)^2 = (2y-1)(2y+1) .
Observe que podemos concluir que 
:
a) Ou 2y-1 = a^2 e 2y+1 = 3b^2

b) Ou 2y-1 = 3c^2 e 2y+1 =
d^2 .

Observe que 3b^2 = a^2
+2 é a única que pode ocorrer
e, como a é ímpar ,
podemos escrever 
a = 2t +1 e 4y = 2(a^2+1)
implicando y = t^2 + (t+1)^2 ,
ok ?
OBS : (1) Esta questão se encontra no
Livro POWER PLAY de
EDWARD J. BARBEAU da MAA ; inclusive com
a solução acima 
(2) O interessante é que para 3x^2+3x
+1 =y^2 tem para solução geral
:
x1 = 4y+7x+3 e y1 = 7y+12x+6
com x e y conhecidos . Exemplo
: x1 = 104 e y1 =181 ; Lindo não é
?

[]´s Carlos Victor


At 20:23 24/1/2006, Klaus Ferraz wrote:
Mostre que a diferença entre os
cubos de dois numeros inteiros consecutivos é igual ao quadrado de um
inteiro, entao esse inteiro é igual a soma dos quadrados de dois inteiros
consecutivos.
Ex: 8^3-7^3=169. 2^2+3^2=13.

Grato.

Yahoo! doce lar.
Faça
do Yahoo! sua homepage. 



Re: [obm-l] Teoria dos Numeros[off - topic]

2006-01-26 Por tôpico Klaus Ferraz
Vlw. Onde consigo esse livro, POWER PLAY de EDWARD J. BARBEAU da MAA Carlos Victor [EMAIL PROTECTED] escreveu:  Olá Klauss ,(x+1)^3 - x^3 = y^2 , onde 3(2x+1)^2 = (2y-1)(2y+1) . Observe que podemos concluir que :a) Ou 2y-1 = a^2 e 2y+1 = 3b^2 b) Ou 2y-1 = 3c^2 e 2y+1 = d^2 .Observe que 3b^2 = a^2 +2 é a única que pode ocorrer e, como a é ímpar , podemos escrever a = 2t +1 e 4y = 2(a^2+1) implicando y
 = t^2 + (t+1)^2 , ok ?OBS : (1) Esta questão se encontra no Livro POWER PLAY de EDWARD J. BARBEAU da MAA ; inclusive com a solução acima (2) O interessante é que para 3x^2+3x +1 =y^2 tem para solução geral :x1 = 4y+7x+3 e y1 = 7y+12x+6 com x e y conhecidos . Exemplo : x1 = 104 e y1 =181 ; Lindo não é ?[]´s Carlos VictorAt 20:23 24/1/2006, Klaus Ferraz wrote:  Mostre que a diferença entre os cubos de dois numeros inteiros consecutivos é igual ao quadrado de um inteiro, entao esse inteiro é igual a soma dos quadrados !
de dois
 inteiros consecutivos.Ex: 8^3-7^3=169. 2^2+3^2=13.Grato.Yahoo! doce lar. Faça do Yahoo! sua homepage. 
		 
Yahoo! doce lar. Faça do Yahoo! sua homepage.

Re: [obm-l] Teoria dos Numeros[off - topic]

2006-01-26 Por tôpico Igor Castro



na www.amazon.com



  - Original Message - 
  From: 
  Klaus 
  Ferraz 
  To: obm-l@mat.puc-rio.br 
  Sent: Thursday, January 26, 2006 7:00 
  PM
  Subject: Re: [obm-l] Teoria dos 
  Numeros[off - topic]
  
  Vlw. Onde consigo esse livro, POWER 
  PLAY de EDWARD J. BARBEAU da 
  MAA Carlos Victor [EMAIL PROTECTED] 
  escreveu: 
  Olá 
Klauss ,(x+1)^3 - x^3 = y^2 , onde 3(2x+1)^2 = 
(2y-1)(2y+1) . Observe que podemos concluir 
que :a) Ou 2y-1 = a^2 e 2y+1 = 
3b^2 b) Ou 2y-1 = 3c^2 e 2y+1 = 
d^2 .Observe que 
3b^2 = a^2 +2 é a única que 
pode ocorrer e, como a é 
ímpar , podemos escrever a = 2t 
+1 e 4y = 2(a^2+1) 
implicando y = t^2 + (t+1)^2 , 
ok ?OBS : (1) Esta questão se 
encontra no Livro POWER PLAY 
de EDWARD J. BARBEAU da MAA ; inclusive 
com a solução acima (2) O 
interessante é que para 3x^2+3x +1 =y^2 
tem para solução geral :x1 = 
4y+7x+3 e y1 = 7y+12x+6 com 
x e y conhecidos . Exemplo : x1 = 104 e 
y1 =181 ; Lindo não é ?[]´s 
Carlos VictorAt 20:23 24/1/2006, Klaus Ferraz 
wrote:
Mostre que a diferença entre os 
  cubos de dois numeros inteiros consecutivos é igual ao quadrado de um 
  inteiro, entao esse inteiro é igual a soma dos quadrados ! de dois 
  inteiros consecutivos.Ex: 8^3-7^3=169. 
  2^2+3^2=13.Grato.Yahoo! doce lar. Faça 
  do Yahoo! sua homepage. 
  
  
  Yahoo! doce lar. Faça 
  do Yahoo! sua homepage.
  
  

  No virus found in this incoming message.Checked by AVG Free 
  Edition.Version: 7.1.375 / Virus Database: 267.14.22/238 - Release Date: 
  23/1/2006


[obm-l] Teoria dos Numeros

2006-01-24 Por tôpico Klaus Ferraz
Mostre que a diferença entreos cubos de doisnumeros inteiros consecutivos é igual ao quadrado de um inteiro, entao esse inteiro é igual a soma dos quadrados de dois inteiros consecutivos.  Ex: 8^3-7^3=169. 2^2+3^2=13.Grato.
		 
Yahoo! doce lar. Faça do Yahoo! sua homepage.

[obm-l] teoria dos numeros!!!

2005-12-29 Por tôpico diego andres
alguem me mostra um jeito fácil de descobrir um sistema completo de residuos  modulo 7onde todos os elementos sao primos...  valeu ai pessoal,Diego
		 
Yahoo! doce lar. Faça do Yahoo! sua homepage.

[obm-l] teoria dos numeros

2005-10-19 Por tôpico Rodrigo Augusto

boa tarde a todos, quem me ajuda com esse?

sabemos que existem infinitos numeros primos da forma 4k + 3. dado um 
inteiro b e sendo S o conjunto de todos os primos da forma p = 4k + 3 , onde 
p não divide b. a questão é: existem dois primos (4k + 3) e (4q + 3) em S de 
tal forma que (k + 1) e (q + 1) são primos entre si?


abraços

_
Chegou o que faltava: MSN Acesso Grátis. Instale Já! 
http://www.msn.com.br/discador


=
Instruções para entrar na lista, sair da lista e usar a lista em
http://www.mat.puc-rio.br/~nicolau/olimp/obm-l.html
=


Re:[obm-l] teoria dos numeros

2005-10-19 Por tôpico claudio\.buffara
Que tal 7 e 19?
7 = 4*1 + 3 e 19 = 4*4 + 3
mdc(1+1,4+1) = 1.

[]s,
Claudio.





De:
[EMAIL PROTECTED]




Para:
obm-l@mat.puc-rio.br




Cópia:





Data:
Wed, 19 Oct 2005 16:01:03 -0200




Assunto:
[obm-l] teoria dos numeros
 boa tarde a todos, quem me ajuda com esse?
 
 sabemos que existem infinitos numeros primos da forma 4k + 3. dado um 
 inteiro b e sendo S o conjunto de todos os primos da forma p = 4k + 3 , onde 
 p não divide b. a questão é: existem dois primos (4k + 3) e (4q + 3) em S de 
 tal forma que (k + 1) e (q + 1) são primos entre si?
 
 abraços
 
 _
 Chegou o que faltava: MSN Acesso Grátis. Instale Já! 
 http://www.msn.com.br/discador
 
 =
 Instruções para entrar na lista, sair da lista e usar a lista em
 http://www.mat.puc-rio.br/~nicolau/olimp/obm-l.html
 =
 


[obm-l] Teoria dos numeros

2005-02-06 Por tôpico kleinad
Aqui vai um probleminha (que eu achei!) legal:

Seja p um número primo. Seja A_d = { a em (Z/pZ)* tal que ord(a) = d } para
cada d divisor de fi(p), onde (Z/pZ)* = (Z/pZ) - { 0 } e fi é a função de
Euler. Definimos f(d) = soma de todos os elementos de A_d. Prove que f(d) ==
mi(d) (mod p) para todo d divisor de fi(p), onde mi é a função de Möbius.

[]s,
Daniel

=
Instruções para entrar na lista, sair da lista e usar a lista em
http://www.mat.puc-rio.br/~nicolau/olimp/obm-l.html
=


[obm-l] Teoria dos Numeros-Soluçao de um Hojoo Lee

2004-05-03 Por tôpico Johann Peter Gustav Lejeune Dirichlet
Ola turma!!!
Como disse o Claudio, vamos nos esbaldar em problemas.TN nao e meu preferido mas...
Acabei de dar uma passada pelo site do Hojoo Lee e fiz esse problema da apostila de TN.Vejam so que legal...



"Seja p um primo impar.
Prove que existem infinitos primos x tais que 2p divide x-1".

SOLUÇAO:

Considere alguns primos da forma 2pk+1, no conjunto{ p_1 ,..., p_n } .
Seja a o produto desses caras, N=a^p-1, e m um caratal que 
(a-1)m=N.
Nao deve ser difícil ver que a=1 (mod p), logo p|m e p|N.
Seja q um fator primo de m com qp. Veja que q nao divide a-1, senao a^L=1 (mod q) para todo L, e ai 0=m=1+1+1+...+1=p (mod q), impossivel.
Apesar disso, q divide a^p-1 e q divide a^(q-1)-1. Um teorema famoso garante que q divide a^(MDC(p;q-1))-1. Mas q nao divide a-1, e assim MDC(p;q-1)1 e ainda por cima MDC(p;q-1) divide p. Logo esse MDC tem que ser p. E assim q=1 (mod p).
Veja que a nao e par e m tambem nao (um numero impar de parcelas impares). E como q divide m, q e impar.Logo q=1 (mod 2p).
E e claro que q nao divide p_i , 1=i=n, pois senao q|N acarreta q|(-1), absurdo!
Assim, dado um conjunto finito de primos da forma 1+2kp, sempre e possivel arranjar mais um que nao esta no conjunto. Essa e uma boa definiçao de infinito, ces nao acham?

E por hoje e so!
Ass.:Johann

TRANSIRE SVVM PECTVS MVNDOQVE POTIRI
CONGREGATI EX TOTO ORBE MATHEMATICI OB SCRIPTA INSIGNIA TRIBVERE
Fields Medal(John Charles Fields)Yahoo! Messenger - Fale com seus amigos online. Instale agora!

[obm-l] Re: [obm-l] Teoria dos Numeros-Soluçao de um Hojoo Lee

2004-05-03 Por tôpico Domingos Jr.
Seja p um primo impar.
Prove que existem infinitos primos x tais que 2p divide x-1.

considere a PA {(2p)n + 1 : n pertence a Z}
como mdc(2p, 1) = 1 temos, pelo seu teorema (Dirichlet) que tal PA possui
infinitos primos.
ou seja, este problema é um caso particular do super-canhão-teorema de PAs.

[ ]'s

=
Instruções para entrar na lista, sair da lista e usar a lista em
http://www.mat.puc-rio.br/~nicolau/olimp/obm-l.html
=


Re: [obm-l] Teoria dos Numeros

2004-04-20 Por tôpico Claudio Buffara
Title: Re: [obm-l] Teoria dos Numeros



on 17.04.04 10:56, Johann Peter Gustav Lejeune Dirichlet at [EMAIL PROTECTED] wrote:

Seja X o conjunto dos primos tais que se a e b sao dois elementos dele entao ab+4 e a^2+4 tambem estao.Prove ou disprove: X e vazio

Alem da solucao do Gugu, existe uma outra, encontrada pelo Carlos Yuzo Shine usando congruencia mod 7.


[]s,
Claudio.





Re: [obm-l] Teoria dos Numeros

2004-04-20 Por tôpico Johann Peter Gustav Lejeune Dirichlet
Pode-se dizer que sim.Eu preferi escrever desse
jeito para nao dar margem a duvidas
 --- Carlos Gustavo Tamm de Araujo Moreira
[EMAIL PROTECTED] escreveu: Caros Claudio e
Dirichlet,
Bacana esse problema. 
Vamos la': Dadas essas condicoes, se a
 pertence a X entao

b(n)=a^(n+2)+4.(a^n+a^(n-1)+...+a+1)=a^(n+2)+4.(a^(n+1)-1)/(a-1)
 pertence a
 X para todo n, mas para todo primo q (digamos
 q=b(0)=a^2+4), b(n) (mod q) e'
 periodica com periodo divisor de q-1, pelo
 pequeno teorema de Fermat, donde,
 em particular, b(a^2+3) e' multiplo de
 b(0)=a^2+4, e como claramente
 b(a^2+3)  a^2+4, b(a^2+3) nao e' primo,
 absurdo. Assim, X tem que ser vazio.
Abracos,
  Gugu
 
 Obs.: A primeira condicao e' um caso particular
 da segunda, nao ?
  
 
 on 19.04.04 12:54, Ricardo Bittencourt at
 [EMAIL PROTECTED] wrote:
 
  Johann Peter Gustav Lejeune Dirichlet wrote:
  
  (a,b = ab+4 e a^2+4)
  Mas espere, 6m-1=m-1 mod 5.Logo
 (6m-1)(6n-1)+4=mn-m-n mod 5.Sera que da
  para arrancar alguem mod 5?
  Se mn=m+n mod 5 entao nao da primo
  
  Eu já consegui mostrar que todos os
 elementos do
  conjunto são da forma 30k+23, mas ainda não
 cheguei em
  nenhuma contradição.
  
 Mais ainda: eles tem que ser da forma 60k +
 53. Infelizmente, eu acho que
 esse caminho nao vai levar aa solucao, mas
 pelo menos aumenta a minha
 conviccao de que X eh vazio.
 
 Alias, uma pergunta pro Dirichlet: de onde
 voce tirou esse problema?
 
 []s,
 Claudio.
 
 

=
 Instruções para entrar na lista, sair da lista
 e usar a lista em

http://www.mat.puc-rio.br/~nicolau/olimp/obm-l.html

=
 
 

=
 Instruções para entrar na lista, sair da lista
 e usar a lista em

http://www.mat.puc-rio.br/~nicolau/olimp/obm-l.html

= 

=

TRANSIRE SVVM PECTVS MVNDOQVE POTIRI

CONGREGATI EX TOTO ORBE MATHEMATICI OB SCRIPTA INSIGNIA TRIBVERE

Fields Medal(John Charles Fields)




__

Yahoo! Messenger - Fale com seus amigos online. Instale agora! 
http://br.download.yahoo.com/messenger/
=
Instruções para entrar na lista, sair da lista e usar a lista em
http://www.mat.puc-rio.br/~nicolau/olimp/obm-l.html
=


Re: [obm-l] Teoria dos Numeros

2004-04-20 Por tôpico Johann Peter Gustav Lejeune Dirichlet
Uma resposta para o Claudio:

Este problema eu propus logo quando eu entrei na
lista.Ninguem tinha mandado nada sobre
isso.resolvi mandar de novo agora que vi em uns
papeis de matematica olimpica que eu guardava.
Eu ate um tempinho atras so tinha limitado os
caras em algumas congruencias,mas nada
conclusivo.


 --- Claudio Buffara
[EMAIL PROTECTED] escreveu:  on
19.04.04 12:54, Ricardo Bittencourt at
 [EMAIL PROTECTED] wrote:
 
  Johann Peter Gustav Lejeune Dirichlet wrote:
  
  (a,b = ab+4 e a^2+4)
  Mas espere, 6m-1=m-1 mod 5.Logo
 (6m-1)(6n-1)+4=mn-m-n mod 5.Sera que da
  para arrancar alguem mod 5?
  Se mn=m+n mod 5 entao nao da primo
  
  Eu já consegui mostrar que todos os elementos
 do
  conjunto são da forma 30k+23, mas ainda não
 cheguei em
  nenhuma contradição.
  
 Mais ainda: eles tem que ser da forma 60k + 53.
 Infelizmente, eu acho que
 esse caminho nao vai levar aa solucao, mas pelo
 menos aumenta a minha
 conviccao de que X eh vazio.
 
 Alias, uma pergunta pro Dirichlet: de onde voce
 tirou esse problema?
 
 []s,
 Claudio.
 
 

=
 Instruções para entrar na lista, sair da lista
 e usar a lista em

http://www.mat.puc-rio.br/~nicolau/olimp/obm-l.html

=r/~nicolau/olimp/obm-l.html

= 

=

TRANSIRE SVVM PECTVS MVNDOQVE POTIRI

CONGREGATI EX TOTO ORBE MATHEMATICI OB SCRIPTA INSIGNIA TRIBVERE

Fields Medal(John Charles Fields)




__

Yahoo! Messenger - Fale com seus amigos online. Instale agora! 
http://br.download.yahoo.com/messenger/
=
Instruções para entrar na lista, sair da lista e usar a lista em
http://www.mat.puc-rio.br/~nicolau/olimp/obm-l.html
=


Re: [obm-l] Teoria dos Numeros

2004-04-20 Por tôpico Johann Peter Gustav Lejeune Dirichlet
Onde esta ela?
Alias sera que da para generalizar esse quatro?

 --- Claudio Buffara
[EMAIL PROTECTED] escreveu:  on
17.04.04 10:56, Johann Peter Gustav Lejeune
 Dirichlet at
 [EMAIL PROTECTED] wrote:
 
 Seja X o conjunto dos primos tais que se a e b
 sao dois elementos dele entao
 ab+4 e a^2+4 tambem estao.Prove ou disprove: X
 e vazio
 
 Alem da solucao do Gugu, existe uma outra,
 encontrada pelo Carlos Yuzo Shine
 usando congruencia mod 7.
 
 
 []s,
 Claudio.
 
  

=

TRANSIRE SVVM PECTVS MVNDOQVE POTIRI

CONGREGATI EX TOTO ORBE MATHEMATICI OB SCRIPTA INSIGNIA TRIBVERE

Fields Medal(John Charles Fields)




__

Yahoo! Messenger - Fale com seus amigos online. Instale agora! 
http://br.download.yahoo.com/messenger/
=
Instruções para entrar na lista, sair da lista e usar a lista em
http://www.mat.puc-rio.br/~nicolau/olimp/obm-l.html
=


Re: [obm-l] Teoria dos Numeros

2004-04-20 Por tôpico Johann Peter Gustav Lejeune Dirichlet
A soluçao do Gugu, como ja era de se esperar, foi
demais!!
 --- Johann Peter Gustav Lejeune Dirichlet
[EMAIL PROTECTED] escreveu:  Uma
resposta para o Claudio:
 
 Este problema eu propus logo quando eu entrei
 na
 lista.Ninguem tinha mandado nada sobre
 isso.resolvi mandar de novo agora que vi em uns
 papeis de matematica olimpica que eu guardava.
 Eu ate um tempinho atras so tinha limitado os
 caras em algumas congruencias,mas nada
 conclusivo.
 
 
  --- Claudio Buffara
 [EMAIL PROTECTED] escreveu:  on
 19.04.04 12:54, Ricardo Bittencourt at
  [EMAIL PROTECTED] wrote:
  
   Johann Peter Gustav Lejeune Dirichlet
 wrote:
   
   (a,b = ab+4 e a^2+4)
   Mas espere, 6m-1=m-1 mod 5.Logo
  (6m-1)(6n-1)+4=mn-m-n mod 5.Sera que da
   para arrancar alguem mod 5?
   Se mn=m+n mod 5 entao nao da primo
   
   Eu já consegui mostrar que todos os
 elementos
  do
   conjunto são da forma 30k+23, mas ainda não
  cheguei em
   nenhuma contradição.
   
  Mais ainda: eles tem que ser da forma 60k +
 53.
  Infelizmente, eu acho que
  esse caminho nao vai levar aa solucao, mas
 pelo
  menos aumenta a minha
  conviccao de que X eh vazio.
  
  Alias, uma pergunta pro Dirichlet: de onde
 voce
  tirou esse problema?
  
  []s,
  Claudio.
  
  
 

=
  Instruções para entrar na lista, sair da
 lista
  e usar a lista em
 

http://www.mat.puc-rio.br/~nicolau/olimp/obm-l.html
 

=r/~nicolau/olimp/obm-l.html
 

=
 
 
 =
 
 TRANSIRE SVVM PECTVS MVNDOQVE POTIRI
 
 CONGREGATI EX TOTO ORBE MATHEMATICI OB SCRIPTA
 INSIGNIA TRIBVERE
 
 Fields Medal(John Charles Fields)
 
 
 
 

__
 
 Yahoo! Messenger - Fale com seus amigos online.
 Instale agora! 
 http://br.download.yahoo.com/messenger/

=
 Instruções para entrar na lista, sair da lista
 e usar a lista em

http://www.mat.puc-rio.br/~nicolau/olimp/obm-l.html

= 

=

TRANSIRE SVVM PECTVS MVNDOQVE POTIRI

CONGREGATI EX TOTO ORBE MATHEMATICI OB SCRIPTA INSIGNIA TRIBVERE

Fields Medal(John Charles Fields)




__

Yahoo! Messenger - Fale com seus amigos online. Instale agora! 
http://br.download.yahoo.com/messenger/
=
Instruções para entrar na lista, sair da lista e usar a lista em
http://www.mat.puc-rio.br/~nicolau/olimp/obm-l.html
=


Re: [obm-l] Teoria dos Numeros

2004-04-20 Por tôpico Claudio Buffara
Analise os 7 casos possiveis, a == 0, 1, 2, 3, 4, 5 e 6 (mod 7). Alias, nem
precisa analisar todos, jah que k^2 == (7-k)^2 (mod 7). A solucao sai
facilmente.



Sobre a generalizacao, suponhamos que a condicao seja:
a, b pertencem a X == ab + k pertence a X, com k = inteiro fixo.

Entao: 
a pertence a X ==
a^(n+2) + k*(a^n + a^(n-1)+...+ a + 1) =
a^(n+2) + k*(a^(n+1) - 1)/(a - 1) pertence a X para todo n.

Definimos a sequencia (b(n)) da seguinte forma:
b(0) = a
b(n) = a*b(n-1) + k, para n = 1.

Eh claro que b(1) = a^2 + k = p  a e tambem que mdc(a,p) = 1 (jah que a e p
sao primos distintos).
Tambem eh claro que b(n) = a^(n+1) + k*(a^n - 1)/(a - 1) pertence a X.

Em particular, b(p-1) = a^p + k*(a^(p-1) - 1)/(a - 1).

Mas, pelo pequeno teorema de Fermat, a^p == a  e  a^(p-1) == 1 (mod p).

Logo, b(p-1) == a = b(0) (mod p) ==
b(p) = a*b(p-1) + k == a*b(0) + k = b(1) = p (mod p) ==

p = b(1) divide b(p) ==
b(p) eh composto ==
contradicao ==
X eh vazio.

Qualquer semelhanca coma demonstracao do Gugu NAO eh mera coincidencia.

[]s,
Claudio.

on 20.04.04 13:46, Johann Peter Gustav Lejeune Dirichlet at
[EMAIL PROTECTED] wrote:

 Onde esta ela?
 Alias sera que da para generalizar esse quatro?
 
 --- Claudio Buffara
 [EMAIL PROTECTED] escreveu:  on
 17.04.04 10:56, Johann Peter Gustav Lejeune
 Dirichlet at
 [EMAIL PROTECTED] wrote:
 
 Seja X o conjunto dos primos tais que se a e b
 sao dois elementos dele entao
 ab+4 e a^2+4 tambem estao.Prove ou disprove: X
 e vazio
 
 Alem da solucao do Gugu, existe uma outra,
 encontrada pelo Carlos Yuzo Shine
 usando congruencia mod 7.
 
 
 []s,
 Claudio.
 
 

=
Instruções para entrar na lista, sair da lista e usar a lista em
http://www.mat.puc-rio.br/~nicolau/olimp/obm-l.html
=


Re: [obm-l] Teoria dos Numeros

2004-04-20 Por tôpico Johann Peter Gustav Lejeune Dirichlet
Qualquer coincidencia e mera semelhança...Claudio Buffara [EMAIL PROTECTED] wrote:
Analise os 7 casos possiveis, a == 0, 1, 2, 3, 4, 5 e 6 (mod 7). Alias, nemprecisa analisar todos, jah que k^2 == (7-k)^2 (mod 7). A solucao saifacilmente.Sobre a generalizacao, suponhamos que a condicao seja:a, b pertencem a X == ab + k pertence a X, com k = inteiro fixo.Entao: a pertence a X ==a^(n+2) + k*(a^n + a^(n-1)+...+ a + 1) =a^(n+2) + k*(a^(n+1) - 1)/(a - 1) pertence a X para todo n.Definimos a sequencia (b(n)) da seguinte forma:b(0) = ab(n) = a*b(n-1) + k, para n = 1.Eh claro que b(1) = a^2 + k = p  a e tambem que mdc(a,p) = 1 (jah que a e psao primos distintos).Tambem eh claro que b(n) = a^(n+1) + k*(a^n - 1)/(a - 1) pertence a X.Em particular, b(p-1) = a^p + k*(a^(p-1) - 1)/(a - 1).Mas, pelo pequeno teorema de Fermat, a^p == a e a^(p-1) == 1 (mod
 p).Logo, b(p-1) == a = b(0) (mod p) ==b(p) = a*b(p-1) + k == a*b(0) + k = b(1) = p (mod p) ==p = b(1) divide b(p) ==b(p) eh composto ==contradicao ==X eh vazio.Qualquer semelhanca coma demonstracao do Gugu NAO eh mera coincidencia.[]s,Claudio.on 20.04.04 13:46, Johann Peter Gustav Lejeune Dirichlet at[EMAIL PROTECTED] wrote: Onde esta ela? Alias sera que da para generalizar esse quatro?  --- Claudio Buffara <[EMAIL PROTECTED]>escreveu:  on 17.04.04 10:56, Johann Peter Gustav Lejeunne Dirichlet at [EMAIL PROTECTED] wrote:  Seja X o conjunto dos primos tais que se a e b sao dois elementos dele entao ab+4 e a^2+4 tambem estao.Prove ou disprove: X e vazio  Alem da solucao do Gugu, existe uma
 outra, encontrada pelo Carlos Yuzo Shine usando congruencia mod 7.   []s, Claudio.  =Instruções para entrar na lista, sair da lista e usar a lista emhttp://www.mat.puc-rio.br/~nicolau/olimp/obm-l.html=

TRANSIRE SVVM PECTVS MVNDOQVE POTIRI
CONGREGATI EX TOTO ORBE MATHEMATICI OB SCRIPTA INSIGNIA TRIBVERE
Fields Medal(John Charles Fields)Yahoo! Messenger - Fale com seus amigos online. Instale agora!

Re: [obm-l] Teoria dos Numeros

2004-04-19 Por tôpico Ricardo Bittencourt
Johann Peter Gustav Lejeune Dirichlet wrote:

(a,b = ab+4 e a^2+4)
Mas espere, 6m-1=m-1 mod 5.Logo (6m-1)(6n-1)+4=mn-m-n mod 5.Sera que da 
para arrancar alguem mod 5?
Se mn=m+n mod 5 entao nao da primo
Eu já consegui mostrar que todos os elementos do
conjunto são da forma 30k+23, mas ainda não cheguei em
nenhuma contradição.

Ricardo Bittencourt   http://www.mundobizarro.tk
[EMAIL PROTECTED]   tenki ga ii kara sanpo shimashou
-- União contra o forward - crie suas proprias piadas --
=
Instruções para entrar na lista, sair da lista e usar a lista em
http://www.mat.puc-rio.br/~nicolau/olimp/obm-l.html
=


Re: [obm-l] Teoria dos Numeros

2004-04-19 Por tôpico Claudio Buffara
on 19.04.04 12:54, Ricardo Bittencourt at [EMAIL PROTECTED] wrote:

 Johann Peter Gustav Lejeune Dirichlet wrote:
 
 (a,b = ab+4 e a^2+4)
 Mas espere, 6m-1=m-1 mod 5.Logo (6m-1)(6n-1)+4=mn-m-n mod 5.Sera que da
 para arrancar alguem mod 5?
 Se mn=m+n mod 5 entao nao da primo
 
 Eu já consegui mostrar que todos os elementos do
 conjunto são da forma 30k+23, mas ainda não cheguei em
 nenhuma contradição.
 
Mais ainda: eles tem que ser da forma 60k + 53. Infelizmente, eu acho que
esse caminho nao vai levar aa solucao, mas pelo menos aumenta a minha
conviccao de que X eh vazio.

Alias, uma pergunta pro Dirichlet: de onde voce tirou esse problema?

[]s,
Claudio.


=
Instruções para entrar na lista, sair da lista e usar a lista em
http://www.mat.puc-rio.br/~nicolau/olimp/obm-l.html
=


Re: [obm-l] Teoria dos Numeros

2004-04-19 Por tôpico Carlos Gustavo Tamm de Araujo Moreira
   Caros Claudio e Dirichlet,
   Bacana esse problema. 
   Vamos la': Dadas essas condicoes, se a pertence a X entao
b(n)=a^(n+2)+4.(a^n+a^(n-1)+...+a+1)=a^(n+2)+4.(a^(n+1)-1)/(a-1) pertence a
X para todo n, mas para todo primo q (digamos q=b(0)=a^2+4), b(n) (mod q) e'
periodica com periodo divisor de q-1, pelo pequeno teorema de Fermat, donde,
em particular, b(a^2+3) e' multiplo de b(0)=a^2+4, e como claramente
b(a^2+3)  a^2+4, b(a^2+3) nao e' primo, absurdo. Assim, X tem que ser vazio.
   Abracos,
 Gugu

Obs.: A primeira condicao e' um caso particular da segunda, nao ?
 

on 19.04.04 12:54, Ricardo Bittencourt at [EMAIL PROTECTED] wrote:

 Johann Peter Gustav Lejeune Dirichlet wrote:
 
 (a,b = ab+4 e a^2+4)
 Mas espere, 6m-1=m-1 mod 5.Logo (6m-1)(6n-1)+4=mn-m-n mod 5.Sera que da
 para arrancar alguem mod 5?
 Se mn=m+n mod 5 entao nao da primo
 
 Eu já consegui mostrar que todos os elementos do
 conjunto são da forma 30k+23, mas ainda não cheguei em
 nenhuma contradição.
 
Mais ainda: eles tem que ser da forma 60k + 53. Infelizmente, eu acho que
esse caminho nao vai levar aa solucao, mas pelo menos aumenta a minha
conviccao de que X eh vazio.

Alias, uma pergunta pro Dirichlet: de onde voce tirou esse problema?

[]s,
Claudio.


=
Instruções para entrar na lista, sair da lista e usar a lista em
http://www.mat.puc-rio.br/~nicolau/olimp/obm-l.html
=


=
Instruções para entrar na lista, sair da lista e usar a lista em
http://www.mat.puc-rio.br/~nicolau/olimp/obm-l.html
=


[obm-l] Teoria dos Numeros

2004-04-17 Por tôpico Johann Peter Gustav Lejeune Dirichlet
Seja X o conjunto dos primos tais que se a e b sao dois elementos dele entao ab+4 e a^2+4 tambem estao.Prove ou disprove: X e vazio

TRANSIRE SVVM PECTVS MVNDOQVE POTIRI
CONGREGATI EX TOTO ORBE MATHEMATICI OB SCRIPTA INSIGNIA TRIBVERE
Fields Medal(John Charles Fields)Yahoo! Messenger - Fale com seus amigos online. Instale agora!

Re: [obm-l] Teoria dos Numeros

2004-04-17 Por tôpico Claudio Buffara
Title: Re: [obm-l] Teoria dos Numeros



on 17.04.04 10:56, Johann Peter Gustav Lejeune Dirichlet at [EMAIL PROTECTED] wrote:

Seja X o conjunto dos primos tais que se a e b sao dois elementos dele entao ab+4 e a^2+4 tambem estao.Prove ou disprove: X e vazio


Inicio de solucao:
 
Suponhamos que X  vazio e seja n um elemento de X.
Eh facil ver que n  2.
Alem disso, n  3, pois se 3 pertencesse a X, teriamos:
3^2 + 4 = 13 em X ==
3*13 + 4 = 43 em X ==
13^2 + 4 = 173 em X ==
43*173 + 4 = 7443 em X ==
contradicao, pois 7443 = 3*2481 eh composto.

Agora, um primo p  3 eh da forma 3k + 1 ou 3k + 2.
Se p = 3k + 1 pertence a X, entao:
p^2 + 4 e p*(p^2 + 4) + 4 = p^3 + 4p + 4 pertencem a X.
Mas p^3 + 4p + 4 = (p^3 - p) + 3*(p + 1) + (2p + 1). 
Alem disso:
p^3 - p eh multiplo de 3, pelo pequeno teorema de Fermat;
3*(p + 1) eh multiplo de 3 (obviamente);
2p + 1 = 2*(3k + 1) + 1 = 3*(2k + 1) eh multiplo de 3.
Ou seja, p*(p^2 + 4) + 4 eh multiplo de 3 e, portanto, composto ==
contradicao ==
nenhum primo da forma 3k + 1 pertence a X.

Em suma, se algum primo pertence a X, ele serah da forma 3k + 2.
Mais ainda: se k for par (por exemplo, k = 2m, com m  0), teremos que:
3k + 2 = 3*(2m) + 2 = 2*(3m + 1) eh par e, portanto, nao pode pertencer a X.

Logo, se algum primo pertence a X, ele serah da forma 3*(2m-1) + 2 = 6m - 1, com m  0. 

De fato, podemos dizer que m  1, pois se 5 pertencesse a X, entao:
5^2 + 4 = 29 pertence a X ==
29^2 + 4 = 845 pertence a X ==
contradicao, pois 845 eh multiplo de 5.

Por enquanto isso foi tudo que eu consegui e suspeito que tenha sido a parte mais facil. Enfim, jah eh um comeco!


[]s,
Claudio. 






[obm-l] teoria dos numeros

2004-01-21 Por tôpico levi queiroz

Pessoal da lista , eu estou enviando para de vocês quatro proposições minhas que eu mesmo demonstrei e no entanto eu não sei se constam dentro da Teoria dos Números. Gostaria da ajuda de vocês.
Proposição 1: Se p 3e p+2 são primos gêmeos então p +1 = 6k, para algum k inteiro
Como pé primo ímpar então p+1 é par, daí 2 divide p+1. Basta provar que 3 divide p+1.
vamos supor que 3 não divide p+1, daí temos duas situações possíveis:

p+1= 3.k +1 para algum k inteiro, daí p = 3.k absurdo pois p é primo 
ou,
p+1 = 3.k +2, para algum k inteiro, então p + 1 + 1 = 3.k + 2 +1, entãop+2 = 3.k +3 = 3( k+1) = 3.m, absurdo pois p+2 é primo.
logo por (a) e (b) temos que 3 divide p+1. Como 2 divide p+1 e 3 divide p+1, então 6 divide p+1, daí p+1= 6.k. Como queríamos demonstrar!
Proposição 2: Sep3 e p+2são primos gêmeos então p= 6.k +5, para algum k inteiro
De fato,
pela proposição 1 temos que : p+1 = 6.m, para algum m inteiro p = 6.m - 1= 6.m + 5 - 6 = 6.( m-1) + 5 = 6.k +5 .
Proposição 3: Se p3e 2.p +1 são primos então p+1=6.n , para algum n inteiro.
Comop é primo ímpar então p+1 é par, daí 2 divide p+1. Basta provar que 3 divide p+1.
vamos supor que 3 não divide p+1, daí temos duas situações possíveis:
a)
p+1= 3.k +1, para algum k inteiro, então p = 3.k absurdo pois p é primo, ou 
p+1 = 3.k +2, par algum k inteiro, então p = 3.k +1, logo 2p = 6.k + 2 , daí2p +1 = 6.k + 3 = 3( 2.k + 1) = 3.m, absurdo pois 2.p +1 é primo por hipótese. 
então por (a) e (b) temos que 3 divide p+1. Como 2 divide p+1, então 6 divide p+1. Daí , p+1 = 6.n, para algum n inteiro.
Proposição 4 : Sep3 e 2p +1 são primos então p= 6.k +5, para algum k inteiro.
Pela proposição 3 temos que : p+1 = 6.m, para algum m inteiro, então p = 6.m - 1= 6.m + 5 - 6 = 6.( m-1) + 5 = 6.k +5, como queríamos demonstrar.
Atenciosamente ,
Levi





Yahoo! GeoCities: a maneira mais fácil de criar seu web site grátis!

Re: [obm-l] teoria dos numeros

2004-01-21 Por tôpico Nicolau C. Saldanha
On Wed, Jan 21, 2004 at 04:46:28PM -0300, levi queiroz wrote:
 Pessoal da lista , eu estou enviando para de vocês quatro proposições minhas
 que eu mesmo demonstrei e no entanto eu não sei se constam dentro da Teoria
 dos Números. Gostaria da ajuda de vocês.
 
 Proposição 1: Se  p 3 e p+2 são primos gêmeos então  p +1 = 6k, para algum k
 inteiro
...
 Proposição 2: Se p3 e  p+2 são primos gêmeos então  p= 6.k +5, para algum k
 inteiro
...
 Proposição 3: Se  p3 e 2.p +1 são primos então  p+1=6.n , para algum n
 inteiro.
...
 Proposição 4 : Se p3 e 2p +1 são primos então  p= 6.k +5, para algum k
 inteiro.

Eu não entendi direito que tipo de ajuda você quer. As proposições estão
corretas, as demonstrações tanto quanto eu verifiquei também estão.
Este assunto é teoria dos números; é isso que você queria perguntar?
Ou talvez você estivesse perguntando se os resultados são conhecidos?
Sim, são bem conhecidos.

Não se sabe por outro lado se existem infinitos pares de primos gêmeos.
Se p é primo e 2p+1 também é primo, então p é chamado um primo
de Sophie Germain. Também não se sabe se existem infinitos primos
de Sophie Germain.

Para saber mais sobre números primos eu recomendo que você dê uma olhada em
http://www.utm.edu/research/primes
ou
http://primes.utm.edu

Para primos gêmeos veja
http://primes.utm.edu/top20/page.php?id=1

Tem uma curiosidade lá sobre o infame bug do pentium.

Para primos de Sophie Germain veja
http://primes.utm.edu/top20/page.php?id=2

[]s, N.
=
Instruções para entrar na lista, sair da lista e usar a lista em
http://www.mat.puc-rio.br/~nicolau/olimp/obm-l.html
=


[obm-l] teoria dos numeros - parte II

2004-01-21 Por tôpico levi queiroz
Obrigado Professor Nicolau Saldanha! Eu queria saber se os resultados eram conhecidos. Eu cheguei a estes resultados sem saber que eles já eram de domínio público. De qualquer maneira para mim foi um grande prazer ter encontrado estes resultados. Foi procurar nas fontes indicadas pelo senhor para ver as provas apresentadas para as proposições que eu mandei para lista.Yahoo! GeoCities: a maneira mais fácil de criar seu web site grátis!

Re: [obm-l] Teoria dos numeros

2003-09-15 Por tôpico Marcelo Souza
a) a(a^2-1)
Se a e impar entao
a^2==1 mod 8
e como (a-1)a(a+1) são tres inteiros consecutivos, temos que 3 tb o divide, 
logo 24 divide o produto

b)
Mesmo esquema
a^2==1 mod 8
b^2==1 mod 8
a^2-b^2==0 mod 8

obs.: Considere a=8k+r onde 0=r8

e eleve ao quadrado para os casos impares (para nao fazer mta conta use ao 
inves de r=5, r=-3 (por ai), que fica bem resumido, dai vc observar que o 
quadrado de um numero inteiro impar e congruente a 1 mod 8).

[]'s, Marcelo.

From: Henrique Patrício Sant'Anna Branco [EMAIL PROTECTED]
Reply-To: [EMAIL PROTECTED]
To: OBM [EMAIL PROTECTED]
Subject: [obm-l] Teoria dos numeros
Date: Sun, 14 Sep 2003 20:37:26 -0300
Prove as seguintes afirmações:
a) Se a é um inteiro ímpar, então 24 divide a*(a^2 - 1)
b) Se a e b são inteiros impares, entao 8 divide a^2 - b^2
No caso do item b) pensei em considerar a = 4k-1 e b = 4k+1. Eu perco a
generalidade se fizer algo assim?
Grato,
Henrique.
=
Instruções para entrar na lista, sair da lista e usar a lista em
http://www.mat.puc-rio.br/~nicolau/olimp/obm-l.html
=
_
MSN Messenger: instale grátis e converse com seus amigos. 
http://messenger.msn.com.br

=
Instruções para entrar na lista, sair da lista e usar a lista em
http://www.mat.puc-rio.br/~nicolau/olimp/obm-l.html
=


[obm-l] Teoria dos numeros

2003-09-14 Por tôpico Henrique Patrício Sant'Anna Branco
Prove as seguintes afirmações:
a) Se a é um inteiro ímpar, então 24 divide a*(a^2 - 1)
b) Se a e b são inteiros impares, entao 8 divide a^2 - b^2
No caso do item b) pensei em considerar a = 4k-1 e b = 4k+1. Eu perco a
generalidade se fizer algo assim?

Grato,
Henrique.

=
Instruções para entrar na lista, sair da lista e usar a lista em
http://www.mat.puc-rio.br/~nicolau/olimp/obm-l.html
=


Re: [obm-l] Teoria dos numeros

2003-09-14 Por tôpico Murilo
a) a*(a^2 - 1) = a*(a-1)*(a+1)=(a+1)*a*(a-1)

Notamos que sao 3 numeros consecutivos, e seja a impar, a-1 e a+1 sao pares
q contem um multiplo de 2 e outro de 4, claramente. E em 3 numeros
consecutivos, a probabilidade de se encontrar um multiplo de 3 eh 100% logo
eh multiplo de 4*3*2=24

b) a^2 - b^2=(a+b)*(a-b)

Novamente, seja a e b dois numeros impares, (a+b) e (a-b) sao pares.
a=2*p+1
b=2*q+1

(a+b)*(a-b) = (2*p + 2*q + 2)*(2*p + 2*q) = 2*(p+q+1)*2*(p+q) =
2*2*(p+q+1)*(p+q)
como (p+q) e (p+q+1) sao consecutivos, um dos dois há de ser par logo
possuem o fator dois.
logo eh multiplo de 2*2*2 = 8

- Original Message -
From: Henrique Patrício Sant'Anna Branco [EMAIL PROTECTED]
To: OBM [EMAIL PROTECTED]
Sent: Sunday, September 14, 2003 8:37 PM
Subject: [obm-l] Teoria dos numeros


 Prove as seguintes afirmações:
 a) Se a é um inteiro ímpar, então 24 divide a*(a^2 - 1)
 b) Se a e b são inteiros impares, entao 8 divide a^2 - b^2
 No caso do item b) pensei em considerar a = 4k-1 e b = 4k+1. Eu perco a
 generalidade se fizer algo assim?

 Grato,
 Henrique.

 =
 Instruções para entrar na lista, sair da lista e usar a lista em
 http://www.mat.puc-rio.br/~nicolau/olimp/obm-l.html
 =


=
Instruções para entrar na lista, sair da lista e usar a lista em
http://www.mat.puc-rio.br/~nicolau/olimp/obm-l.html
=


Re: [obm-l] Teoria dos numeros

2003-09-14 Por tôpico Claudio Buffara
on 14.09.03 20:37, Henrique Patrício Sant'Anna Branco at
[EMAIL PROTECTED] wrote:

 Prove as seguintes afirmações:
 a) Se a é um inteiro ímpar, então 24 divide a*(a^2 - 1)
 b) Se a e b são inteiros impares, entao 8 divide a^2 - b^2
 No caso do item b) pensei em considerar a = 4k-1 e b = 4k+1. Eu perco a
 generalidade se fizer algo assim?
 
Infelizmente voce perde, pois poderia ser a = 4k+1, por exemplo.

a) a impar ==
a = 2k+1 para algum inteiro k ==
a(a^2-1) = (2k+1)(4k^2+4k) = 4k(k+1)(2k+1)

Agora voce raciocina assim:
k e k+1 sao inteiros consecutivos ==
um deles eh par ==
2 divide k(k+1) ==
8 divide 4k(k+1)(2k+1)  (*)

Se 3 divide k ou 3 divide k+1, entao 3 divide 4k(k+1)(2k+1) ==
juntamente com (*) isso implica que 24 (=8*3) divide 4k(k+1)(2k+1)

Se 3 nao divide k nem k+1, entao k = 3m+1, para algum inteiro m ==
2k+1 = 6m+3 ==
3 divide 2k+1 ==
3 divide 4k(k+1)(2k+1) ==
juntamente com (*) isso implica que 24 divide 4k(k+1)(2k+1)

*

b) Na verdade, isso eh decorrencia do fato de que se a eh impar entao 8
divide a^2 - 1, pois a = 2m + 1 ==
a^2 - 1 = 4m^2 + 4m + 1 - 1 = 4m(m+1)

Mas, como visto acima, 2 divide m(m+1) ==
8 divide 4m(m+1) = a^2 - 1.


Um abraco,
Claudio.
a^2 - b^2 = 4m^2 + 4m - 4n^2 - 4n = 4[m(m+1) - n(n+1)]

=
Instruções para entrar na lista, sair da lista e usar a lista em
http://www.mat.puc-rio.br/~nicolau/olimp/obm-l.html
=


Re: [obm-l] Teoria dos Numeros

2002-07-15 Por tôpico Nicolau C. Saldanha

On Sun, Jul 14, 2002 at 04:45:19PM -0300, adr.scr.m wrote:
 Alguem poderia fazer essas questoes para  
 mim ?
 
 Determine todos os primos que sao a soma e  
 a diferenca de 2 primos.

5 = 3 + 2 = 7 - 2 'e o 'unico.
Basta observar que 'e indispens'avel usar o primo 2
e que o 'unico caso em que p-2, p e p+2 s~ao todos primos 'e para p=5
pois um dos tr^es sempre ser'a m'ultiplo de 3.

[]s, N.
=
Instruções para entrar na lista, sair da lista e usar a lista em
http://www.mat.puc-rio.br/~nicolau/olimp/obm-l.html
O administrador desta lista é [EMAIL PROTECTED]
=



Re: [obm-l] Teoria dos Numeros

2002-07-15 Por tôpico Nicolau C. Saldanha

On Sun, Jul 14, 2002 at 04:45:19PM -0300, adr.scr.m wrote:
 Determine todos inteiros positivos 
 x,y,z,tais  que z divide xy-1,x divide zy-1
 e y divide zx-1.

Este problema 'e bem legal. Vou pular umas linhas antes de dar a solu,c~ao
para que os outros tentem fazer sozinhos, vale a pena.

==







































==

Uma solu,c~ao trivial 'e x=y=z=1. Qualquer outra solu,c~ao pode ser tomada
da forma 1xyz, x,y e z primos entre si. Temos

xy = 1+az
xz = 1+by
yz = 1+cx

A partir da primeira equa,c~ao temos z=(xy-1)/a e substituindo na 2a eq
temos (x^2 - ab)y = x+a. Como da 1a equa,c~ao devemos ter a = x segue
que 0  x+a  2y donde y = x+a e x^2 - ab = 1. Segue dai que z = x+b
donde

(x+a)(x+b) = 1+cx

ou

x^2 + ax + bx + ab = 1 + cx
(2x + a + b - c) x = 2

e a 'unica solu,c~ao n~ao trivial 'e x=2,y=3,z=5.

[]s, N.

=
Instruções para entrar na lista, sair da lista e usar a lista em
http://www.mat.puc-rio.br/~nicolau/olimp/obm-l.html
O administrador desta lista é [EMAIL PROTECTED]
=



[obm-l] Teoria dos Numeros

2002-07-14 Por tôpico adr.scr.m

Alguem poderia fazer essas questoes para  
mim ?

Determine todos os primos que sao a soma e  
a diferenca de 2 primos.

Determine todos inteiros positivos 
x,y,z,tais  que z divide xy-1,x divide zy-1
e y divide zx-1.

Obrigado.
Adriano.

 
__
AcessoBOL, só R$ 9,90! O menor preço do mercado!
Assine já! http://www.bol.com.br/acessobol


=
Instruções para entrar na lista, sair da lista e usar a lista em
http://www.mat.puc-rio.br/~nicolau/olimp/obm-l.html
O administrador desta lista é [EMAIL PROTECTED]
=



[obm-l] Teoria dos Numeros

2002-03-06 Por tôpico osvaldo correa do nascimento junior

Olá lista,

Desculpe-me o assunto off-topic mas, alguem da lista 
teria o livro de Teoria Elementar dos Numeros de edgard 
de Alencar Filho. Valeu !!!

Osvaldo Correa
=
Instruções para entrar na lista, sair da lista e usar a lista em
http://www.mat.puc-rio.br/~nicolau/olimp/obm-l.html
O administrador desta lista é [EMAIL PROTECTED]
=